You are on page 1of 75

Ketaksamaan

Ajat Adriansyah

August 31, 2006


ii

Pengantar
Tulisan ini lebih banyak menekankan pada materi, untuk lebih mahir dalam
pengerjaan soal, pembaca diharapkan untuk mencari soal-soal sebanyak mungkin
untuk dikerjakan. Apabila terdapat kesalahan dalam penulisan buku ini, harap
dilaporkan ke ajat.adriansyah@gmail.com.
Ucapan terima kasih untuk: Fajar Yuliawan, Dimas Yusuf W, Aleams barra,
dan Albert Gunawan.
Daftar isi

1 Ketaksamaan Rataan 1
1.1 Ketaksamaan dasar . . . . . . . . . . . . . . . . . . . . . . . . . . 1
1.2 * Pasangan terurut dan Permutasi . . . . . . . . . . . . . . . . . 4
1.3 Ketaksamaan Rataan Aritmatik - Rataan Geometrik . . . . . . . 5
1.4 Ketaksamaan Akar Rataan -AM . . . . . . . . . . . . . . . . . . 12
1.5 Ketaksamaan Jensen . . . . . . . . . . . . . . . . . . . . . . . . . 15

2 Ketaksamaan Norm 25
2.1 Ketaksamaan Cauchy-Schwarz (CS), Minkowski dan Hölder . . 25
2.2 Rearrangement & Ketaksamaan Chebishev . . . . . . . . . . . . 34

3 Penjumlahan Simetrik dan Siklis 47


3.1 Normalisasi dan Subtitusi . . . . . . . . . . . . . . . . . . . . . . 47
3.2 Schur & Muirhead . . . . . . . . . . . . . . . . . . . . . . . . . . 55

4 Mixing Variabel dan Metode Kalkulus 63


4.1 Referensi . . . . . . . . . . . . . . . . . . . . . . . . . . . . . . . 71

iii
iv DAFTAR ISI
Bab 1

Ketaksamaan Rataan

1.1 Ketaksamaan dasar


Ketaksamaan yang akan diperkenalkan pertama kali ini merupakan salah satu
jenis ketaksamaan yang paling dasar dan sering disebut trivial atau cuma −
cuma yaitu bahwa kuadrat suatu bilangan rill selalu lebih besar dari nol. Secara
matematis ditulis untuk setiap x bilangan riil berlaku x2 ≥ 0, Kesamaan terjadi
jika dan hanya jika x = 0. Ketaksamaan ini akan sangat membantu dalam
membuktikan ketaksamaan lain.

Contoh 1.1.1 Tentukan Nilai minimum dari x2 − 12x + 6, kapankah nilai min-
imum tersebut tercapai!

Dengan memodifikasi sedikit x2 − 12x + 6 , akan diperoleh bahwa x2 − 12x + 6 =


x2 − 12x + 36 − 30 = (x − 6)2 − 30 ≥ −30. Jadi nilai minimum dari fungsi diatas
adalah -30. Nilai tersebut akan tercapai jika x − 6 = 0 atau x = 6 .

Contoh 1.1.2 Buktikan ketaksamaan nilai tengah aritmatik - geometrik (AM-


GM) untuk dua variabel yaitu

a+b √
∀ a, b ≥ 0, ≥ ab
2
dengan kasamaan tercapai untuk a = b .
√ √ √
Solusi Berdasarkan ketaksamaan trivial ( a − b)2 ≥ 0, jadi a + b − 2 ab ≥
a+b √
0⇒ ≥ ab. Jika a = b maka kita peroleh kesamaan, jika
2
a+b √
= ab
2

maka a2 − 2ab + b2 = 0 dan diperoleh a = b.

1
2 BAB 1. KETAKSAMAAN RATAAN

Contoh 1.1.3 Buktikan ketaksamaan nilai tengah aritmatik - geometrik (AM-


GM) untuk tiga variabel yaitu
a+b+c √
3
∀ a, b ≥ 0, ≥ abc
3
dengan kasamaan tercapai untuk a = b = c .

Solusi Untuk setiap a, b, c bilangan riil positif berlaku


1
(x + y + z)[(x − y)2 + (x − z)2 + (y − z)2 ] ≥ 0
2
yang akan terjadi samadengan jika dan hanya jika x = y = z, dengan ekspansi
kita peroleh
x3 + y 3 + z 3 ≥ 3xyz
√ √ √
Subtitusikan x = 3 a , y = 3 b dan z = 3 c kita peroleh
a+b+c √
3
≥ abc.
3
Ketaksamaan trivial tampak begitu sederhana, tapi ketaksamaan ini meru-
pakan induk dari ketaksamaan lain. Beberapa ketaksamaan yang akan dibahas
di Bab berikutnya melibatkan ketaksamaan trivial pada penurunannya. Bahkan
banyak soal yang sangat susah yang gagal diselesaikan dengan metode - metode
pada bab berikutnya tapi dapat diselesaikan dengan mengubahnya ke bentuk
(E(a, b, c))2 ≥ 0.
Walaupun diperkenalkan lebih awal, bukan berarti ketaksamaan ini adalah
yang paling mudah digunakan, kemampuan kita dalam menggunakan identi-
tas sangat berpengaruh dalam menyelesaikan soal pertidaksamaan dengan cara
mengubahnya menjadi bentuk kuadrat.
Berikut ini diberikan sebuah contoh soal dari seleksi tim olimpiade Cina
2005, yang juga muncul pada [ONI]1 beserta solusinya yang begitu elegan untuk
soal sesulit ini. Soal ini termasuk soal susah bahkan bagi yang sudah mempela-
jari materi-materi pada subbab berikut.

Contoh 1.1.4 (China 2005 , Vasile Cirtoaje) Diberikn bilangan riil positif
a, b, c, d dengan abcd = 1. Buktikan
1 1 1 1
2
+ 2
+ 2
+ ≥1
(a + 1) (b + 1) (c + 1) (d + 1)2
Solusi Pertama - tama kita mulai dengan membuktikan lemmma berikut
Lemma Jika x dan y adalah bilangan riil positif, maka berlaku
1 1 1
2
+ 2

(1 + x) (1 + y) 1 + xy
1 Old and New Inequalities oleh Vasile Cirtoaje, Mircea Lascu, Titu Andrescu dan Gabriel

Dospinescu. Gil Publisher


1.1. KETAKSAMAAN DASAR 3

Bukti Lemma Perhatikan bahwa


1 1 1 xy(x − y)2 + (xy − 1)2
2
+ 2
− = ≥0
(1 + x) (1 + y) 1 + xy (1 + x)2 (1 + y 2 )(1 + xy)

Jadi lemma telah terbukti.


Dengan Lemma tersebut kita dapatkan
1 1 1 1 1 1
+ ≥ dan + ≥
(1 + a)2 (1 + b)2 1 + ab (1 + c)2 (1 + d)2 1 + cd

Jumlahkan kedua ketaksamaan kita dapatkan


1 1 1 1 1 1
+ + + ≥ +
(a + 1)2 (b + 1)2 (c + 1)2 (d + 1)2 1 + ab 1 + cd
1 + ab + cd + 1
=
1 + ab + cd + abcd
= 1

karena abcd = 1.

Kebanyakan orang mengatakan suatu soal pertidaksamaan itu mudah sete-


lah mereka melihat solusinya, tapi perlu ditekankan bahwa mencari sebuah so-
lusi jauh lebih sulit daripada memahaminya. Solusi dari contoh diatas walaupun
singkat dan terlihat sederhana tapi sulit untuk ditemukan. Ketaksamaan pada
lemma yang merupakan kunci dari solusi diatas tidak mudah untuk terpikirkan.
Tapi para pembaca tidak harus terpaku pada satu solusi dari satu soal, sebuah
masalah dapat mempunyai banyak solusi dan sangat penting bagi kita untuk
mempelajari atau menemukan solusi lain dari suatu masalah yang telah disele-
saikan. Oleh karena itu akan banyak penyelesaian contoh - contoh pada buku ini
diberikan lebih dari satu solusi, walaupun beberapa diantaranya lebih panjang
dan rumit tapi diaharapkan dapat menambah kreativitas berpikir.
Ketaksamaan berikutnya merupakan suatu jenis ketaksamaan geometri non-
trivial yang pertama yaitu ketaksamaan segitiga yang menyatakan jumlah pan-
jang dua sisi segitiga lebih besar dari sisinya yang lain.
Teorema 1.1.1 Jika x dan y adalah dua buah bilangan riil maka berlaku

|x + y| ≤ |x| + |y|

Hal ini juga mengidentifikasikan bahwa pada segitiga dengan panjang sisi a, b
dan c berlaku
a+b>c a+c>b b+c>a
Ditambah lagi jika a, b, c adalah panjang sisi-sisi segitiga, maka dapat meng-
gunakan subtitusi a = x + y, b = y + z dan c = x + z untuk bilangan riil x,
y, z. Subtitusi ini dinamakan subtitusi Ravi yang diambil dari nama Ravi se-
orang kontestan Olimpiade yang pertama kali menggunakannya pada kompetisi.
4 BAB 1. KETAKSAMAAN RATAAN

Latihan 1
1. Berapakah nilai minimum dari cos 2θ − 13 !
2. Buktikan 2(x2 + y 2 ) ≥ (x + y)2 untuk x, y bilangan riil positif.
3. Jika a, b, c, d adalah bilangan riil berapakah nilai minimum dari

a2 + b2 + c2 + d2 + e2 = 2a − 7b + 4c + 3d − 11e + 23

4. Jika a , b dan c adalah bilangan riil positif yang memenuhi a + b + c = 3.


Buktikan
ab bc ac 3
2
+ 2 + 2 ≤
a +1 b +1 c +1 2
5. (Albert Gunawan) Jika a, b, c adalah bilangan riil positif yang mememnuhi
ab + ac + bc = 12, buktikan

abc(3a + b + c)(a + 3b + c)(a + b + 3c) ≥ (12a − bc)(12b − ac)(12c − ab)

Petunjuk: Buktikan bahwa 3a2 + ab + ac ≥ 12a − bc.


6. Untuk setiap a, b, c bilangan riil buktikan bahwa
4 3
(a2 + b2 )2 + (a2 + c2 )2 + (b2 + c2 )2 ≥ (a + b3 + c3 )(a + b + c)
3

1.2 * Pasangan terurut dan Permutasi


Pada subbab ini akan diperkenalkan sebuah konsep dasar dalam matematika
yaitu pasangan terurut, tujuannya adalah untuk memudahkan penuliskan so-
lusi - solusi kita dengan notasi matematika yang umum. Bagi para pembaca
yang sudah familiar dengan subjek ini dapat segera melanjutkan ke subbab
berikutnya.
Definisi Misalkan A dan B adalah dua himpunan tak kosong. Maka hasil kali
silang A × B didefinisikan sebagai

A × B = {(a, b)|a ∈ A dan b ∈ B}.

Lebih umum lagi jika Ai untuk i = 1, 2, · · · , n adalah n buah himpunan tak


kosong maka definisikan

A1 × A2 × · · · × An = {(a1 , a2 , · · · , a2 )|ai ∈ Ai }

Contoh 1.2.1 Jika A = {1, 2, 3} dan B = {2, 3} maka

A × B = {(1, 2), (1, 3), (2, 2), (2, 3), (3, 2), (3, 3)}

Definisi Misalkan Ai untuk i = 1, 2, · · · , n adalah n buah himpunan tak kosong,


dan juga misalkan himpunan O = A1 ×A2 ×· · ·×An , jika u ∈ O maka u disebut
n-tupel atau pasangan terurut, ditambah lagi

u = (a1 , a2 , · · · , a2 ) ai ∈ Ai .
1.3. KETAKSAMAAN RATAAN ARITMATIK - RATAAN GEOMETRIK 5

Definisi Misalkan u = (a1 , a2 , · · · , an ) dan v = (b1 , b2 , · · · , bn ) adalah sebarang


pasangan terurut, kita katakan u = v jika dan hanya jika ai = bi untuk setiap
i = 1, 2, · · · , n.

Definisi Misalkan u = (a1 , a2 , · · · , an ) dan v = (b1 , b2 , · · · , bn ) adalah sebarang


pasangan terurut, maka produk skalar dari u dan v didefinisikan sebagai

u  v = a1 b1 + a2 b2 + · · · + an bn

sedangkan norm-p dari u didefinisikan sebagai


p
||u||p = p (u)  (u)  · · ·  (u)
| {z }
p buah

jika p = 2 maka norm-2 dari u cukup ditulis ||u||.

Definisi Misalkan u = (a1 , a2 , · · · , an ) adalah sebarang pasangan terurut, per-


mutasi dari u adalah semua pasangan terurut yang dapat diperoleh melalui
penukaran posisi dari ai suatu dengan aj .

Contoh 1.2.2 Permutasi dari pasangan terurut (1, 2, 3) adalah

(1, 3, 2) (1, 2, 3) (2, 3, 1) (2, 1, 3) (3, 2, 1) (3, 1, 2)

Definisi Misalkan u = (a1 , a2 , · · · , an ) adalah sebarang pasangan terurut, per-


mutasi siklis dari u adalah semua pasangan terurut yang bebentuk

(a1 , a2 , · · · , an ) (an , a1 , · · · , an−1 ) ··· (a3 , a4 , · · · , a2 ) (a2 , a3 , · · · , a1 ).

1.3 Ketaksamaan Rataan Aritmatik - Rataan


Geometrik
Sebagian besar orang yang mulai untuk belajar pertidaksamaan memulai dengan
belajar ketaksamaan Arithmatic Mean-Geometric Mean, atau disingkat AM-
GM. Banyak soal - soal susah yang dapat diselesaikan secara elegan dengan
AM-GM.
Rataan Aritmatik atau Arithmatic Mean didefinisikan sebagai a1 +a2 +···+a n
√ n
dan rataan geometri atau Geometric mean didefinisikan sebagai n a1 a2 · · · an .
Sedangkan ketaksamaan AM-GM dinyatakan didalam teorema berikut

Teorema 1.3.1 Jika a1 , a2 , · · · , an adalah bilangan - bilangan riil positif, maka


berlaku
a1 + a2 + · · · + an √
≥ n a1 a2 · · · an
n
dengan kesamaan terjadi jika dan hanya jika a1 = a2 = · · · = an
6 BAB 1. KETAKSAMAAN RATAAN

Bukti Kita akan gunakan induksi matematika, untuk kasus n = 2, kita mem-
peroleh bentuk
a+b √
≥ ab
2
√ √
yang benar karena setara dengan ( a − b)2 ≥ 0. Misalkan
a1 + a2 + · · · + an+1 √
An+1 = Gn = n+1
a1 a2 · · · an+1
n+1
Dengan asumsi bahwa AM-GM benar untuk n buah variabel (hipotesis induksi)
kita peroleh
n−1 buah
z }| {
an+1 + (n − 1)An+1 an+1 + An+1 + · · · + An+1 p
= ≥ n an+1 (An+1 )n−1
n n
an+1 + (n − 1)An+1 p
Dengan memisalkan A = dan G = n an+1 (An+1 )n−1 maka
n
An + A
An+1 = ≥ (An A)1/2
2
≥ (Gn G)1/2
n−1 1/n 1/2
= (Gn (an+1 An+1 ) )
= (Gn+1 n−1 1/2n
n+1 An+1 )

Yang berarti (An )2n ≥ Gn+1 n−1


n+1 An+1 =⇒ An+1 ≥ Gn+1 .
Sekarang akan ditunjukkan bahwa kesamaan terjadi jika dan hanya jika
semua ai sama √ i = 1, 2, · · · , n. Untuk n = 2, dapat dilihat bahwa
√ untuk
a = b ⇐⇒ ( a − b)2 = 0. Jika semua ai sama untuk i = 1, 2, · · · , n + 1
maka jelas bahwa An+1 = Gn+1 . Sekarang misalkan An+1 = Gn+1 , maka dari
rantai ketaksamaan diatas kita peroleh
An = A An = Gn A=G
Karena An = Gn maka berdasarkan hipotesis induksi a1 = a2 = · · · = an . Lalu
dari A = G diperoleh
a1 + a2 + · · · + an + 1 nan + an+1
an+1 = An+1 = =
n+1 n+1
jadi an = an+1 , melengkapi bukti kita.
Berikutnya akan diberikan beberapa contoh untuk mengilustrasikan peng-
gunaannya.
Contoh 1.3.1 (Mandelbrot Problem Corner, Thomas J. Mildorf )
Misalkan a , b, dan c adalah bilangan riil positif, tentukan nilai minimum yang
mungkin dari
a3 b 1+c
+ +
4b 8c2 2a
dan tentukan nilai a, b dan c pada saat nilai minimum tersebut tercapai.
1.3. KETAKSAMAAN RATAAN ARITMATIK - RATAAN GEOMETRIK 7

Solusi Perhatikan bahwa dengan aplikasi AM-GM


r
a3 b 1+c a3 b 1 c c 5 a3 · b · 1 · c · c 5
+ 2+ = + 2+ + + ≥5 =
4b 8c 2a 4b 8c 2a 4a 4a 4b · 8c2 · 2a · 4a · 4a 4
Karena kesamaan terjadi jika dan hanya jika

a3 b 1 c c
= 2 = = = ⇐⇒ a = 2, b = 8, c = 2.
4b 8c 2a 4a 4a
Contoh 1.3.2 (International Math Olympiad 2001) Buktikan bahwa

a b c
√ +√ +√ ≥1
a2 + 8bc 2
b + 8ac 2
c + 8ab

untuk a, b dan c bilangan riil positif.

Solusi Kita akan buktikan bahwa

a a4/3
√ ≥ 4/3
a2 + 8bc a + b4/3 + c4/3

Perhatikan bahwa dengan AM-GM kita peroleh

(a4/3 + b4/3 + c4/3 )2 − (a4/3 )2 = (b4/3 + c4/3 )(a4/3 + a4/3 + b4/3 + c4/3 )
≥ 2(bc)2/3 4a2/3 (bc)1/3
= 8a2/3 bc

Jadi

a a4/3
(a4/3 + b4/3 + c4/3 )2 ≥ (a4/3 )2 + 8a2/3 bc ⇐⇒ √ ≥ 4/3
a2 + 8bc a + b4/3 + c4/3

Dengan cara yang sama diperoleh

b b4/3 c b4/3
√ ≥ 4/3 √ ≥ 4/3
b2 + 8ac a + b4/3 + c4/3 c2 + 8ab a + b4/3 + c4/3

Jadi apabila semua hasil diatas ditambahkan diperoleh

a b c a4/3
√ +√ +√ ≥ 4/3 +
a2 + 8bc b2 + 8ac c2 + 8ab a + b4/3 + c4/3
a4/3 a4/3
+
a4/3 + b4/3 + c4/3 a4/3 + b4/3 + c4/3
=1

melengkapi solusi.
8 BAB 1. KETAKSAMAAN RATAAN

Mungkin pada awalnya sulit bagi kita untuk mendapatkan ide awal dari
solusi soal IMO 2001 diatas, angka 4/3 tampak tidak alami untuk ditemukan.
Solusi lain dari masalah ini yang tidak kalah elegan akan diberikan nanti. Untuk
melihat bagaimana ide 4/3 itu diperoleh, perhatikan bahwa pertama - tama kita
mencoba mencari r sehingga
a ar
√ ≥ r ⇐⇒ (ar + br + cr )2 ≥ a2r + 8a2r−2 bc
a2 + 8bc a + br + cr
Yang setara dengan

(ar + br + cr )2 − a2r = (br + cr )(ar + ar + br + cr ) ≥ 8a2r−2 bc

Tapi dengan AM-GM kita peroleh

(br + cr )(ar + ar + br + cr ) ≥ 8ar/2 (bc)3r/4

Ini berarti nilai r yang mungkin harus memenuhi 2r = 2r − 2 dan 3r 4 = 1 yaitu


r = 4/3.
Misalkan pada AM-GM kita mengganti a1 , a2 , · · · , an dengan a11 , a12 , · · · , a1n
maka kita dapatkan
1 1 1
a1 + a2 + · · · + an
r
1 1 1
≥ n ···
n a1 a2 an
yang setara dengan
√ n
n
a1 a2 · · · an ≥ 1 1 1
a1 + a2 + ··· + an

n
perhatikan bahwa bentuk 1
+ a1 +···+ a1n
sering disebut rataan harmonik atau
a1 2
Harmonic Mean (HM) , karena AM ≥ GM dan GM ≥ HM maka juga berlaku
AM ≥ HM , kita nyatakan ketaksamaan GM -HM ini yang telah duperoleh
diatas.
Teorema 1.3.2 Jika a1 , a2 , · · · , an adalah bilangan - bilangan riil positif, maka
berlaku √ n
n
a1 a2 · · · an ≥ 1 1
a1 + a2 + · · · + a1n
dengan kesamaan terjadi jika dan hanya jika a1 = a2 = · · · = an .
Misalkan kita diminta untuk membuktikan A ≥ B, sedangkan dengan men-
gaplikasikan suatu teorema, yang kita dapatkan adalah A ≥ C, pembuktian se-
lesai jika kita berhasil membuktikan C ≥ B. Namun, jika ternyata yang benar
adalah B ≤ C, apa yang harus kita lakukan? Kita dapat menggunakan teorema
lain, atau tetap dengan teorema yang sama tapi diterapkan dengan cara yang
berbeda untuk memperoleh A ≥ D dan D ≥ B. Jadi dalam mengerjakan soal
pertidaksamaan, kita tidak hanya bergantung pada alat - alat yang digunakan,
tapi juga yang terpenting adalah bagaimana cara menerapkan teorema-teorema
1.3. KETAKSAMAAN RATAAN ARITMATIK - RATAAN GEOMETRIK 9

tersebut untuk mendapatkan jalan yang benar menuju solusi, sebagai contoh
untuk membuktikan
a b c a b c
+ 2 + 2 ≤ + +
a2 +2 b +2 c +2 2a + 1 2b + 1 2c + 1

memang dapat dilakukan tanpa perlu menggunakan AM-GM 2 . Tapi jika kita
akan menggunakan AM-GM untuk menyelesaikan soal ini, maka harus diper-
hatikan bagaimana cara menerapkan AM-GM di soal ini. Misalkan AM-GM
diterapkan pada penyebutnya dengan cara berikut

a2 + 2 √ a 1
≥ 2a ⇐⇒ 2 ≤ √
2 a +2 2 2
dan kemudian melakukan cara serupa pada variabel b dan c, maka kita akan
memperoleh

a b c 1 1 1 3
+ + ≤ √ + √ + √ = √
a2 + 2 b2 + 2 c2 + 2 2 2 2 2 2 2 2 2
Berikutnya kita harus membuktikan

3 a b c
√ ≤ + +
2 2 2a + 1 2b + 1 2c + 1

tapi ternyata ketaksamaan


√ diatas salah, karena untuk a = b = c = 1 kita
dapatkan 3 ≤ 2 2 yang jelas salah. Jadi penerapan AM-GM dengan cara diatas
tidak menyelesaikan soal ini, namun ini bukan berarti AM-GM tidak dapat
diterapkan pada soal ini. Kita masih dapat menggunakan AM-GM dengan cara
berikut
a a
a2 + 2 = (a2 + 1) + 1 ≥ 2a + 1 ⇐⇒ 2 ≤
a +2 2a + 1
b b
b2 + 2 = (b2 + 1) + 1 ≥ 2b + 1 ⇐⇒ ≤
+2 b2
2b + 1
c c
c2 + 2 = (c2 + 1) + 1 ≥ 2c + 1 ⇐⇒ 2 ≤
c +2 2c + 1
Yang apabila semua ditambahkan kita peroleh ketaksamaan pada soal.
Sebenarnya pemilihan cara penggunaan AM-GM yang tepat seperti diatas
didasarkan oleh sesuatu, yaitu kasus kesamaan. Perhatikan bahwa pada soal
tersebut kesamaan terjadi jika a = b = c = 1. Jadi jika kita ingin melakukan
AM-GM kita harus melakukannya sedemikian sehingga kita dapatkan kasus
a = 1 sebagai kasus kesamaan. Penerapan AM-GM seperti (a2 + 1) + 1 ≥ 2a + 1
2
menghasilkan
√ a = 1 sebagai 2kasus kesamaan sedangkan penerapan AM-GM
2
a + 2 ≥ 2 2a menghasilkan a = 2 sebagai kasus samadengan. Hasil observasi
pada kasus samadengan dapat menjadi langkah awal untuk menyelesaikan soal
pertidaksamaan, bukan hanya pada soal ini.
2 perhatikan a a
bahwa a2 +2
≤ 2a+1
⇐⇒ (a − 1)2 ≥ 0
10 BAB 1. KETAKSAMAAN RATAAN

Contoh 1.3.3 (Fajar Yuliawan) Jika a, b, c, d > 0 dan abcd = 1, buktikan


1 1 1 1 1
+ + + ≤
a+b+c+9 b+c+d+9 c+d+a+9 d+a+b+9 3
Solusi (Oleh Hendrata Dermawan)
Dengan menggunakan AM-HM kita dapatkan
(a + 3) + (b + 3) + (c + 3) 3
≥ 1 1 1
3 a+3 + b+3 + c+3

yang setara dengan


 
1 1 1 1 1
+ + ≥
9 a+3 b+3 c+3 a+b+c+9
1 1 1
lakukan hal yang serupa dengan b+c+d+9 , c+d+a+9 dan d+c+b+9 . Jadi kita
dapatkan
1 1 1 1
+ + + ≤
a+b+c+9 b+c+d+9 c+d+a+9 d+a+b+9
 
1 1 1 1 1
+ + +
3 a+3 b+3 c+3 d+3
Sehingga cukup bagi kita untuk membuktikan
1 1 1 1
+ + + ≤1
a+3 b+3 c+3 d+3
Dengan mengkalikan silang dan mengeskpansi kita peroleh
108 + 6S2 + 27S1 ≤ abcd + 3S3 + 9S2 + 27S1 + 81 ⇐⇒ 2S3 + 3S2 + 1 ≥ 27
dengan S1 = a + b + c + d , S2 = ab + ac + bc + cd + ad + bd dan S3 =
abc + bcd + cda + dab. Perhatikan bahwa S3 mempunyai 4 suku, S2 mempunyai
6 suku, jadi 2S3 +3S2 +1 mempunyai total 27, dan dengan AM-GM kita peroleh
p
2S3 + 3S2 + 1 ≥ 27 27 (abcd)15
= 27
Para pembaca dapat memeriksa bahwa pada penggunaan AM-HM dan AM-
GM pada contoh diatas, kasus samadengan yaitu a = b = c = d = 1 terawetkan.
Hal ini setidaknya memberikan petunjuk bahwa kita berada pada jalur pemec-
ahan masalah yang tidak salah.
Contoh 1.3.4 (Vietnam 1998) Misalkan x1 , x2 , · · · , xn (n ≥ 2) adalah bi-
langan riil positif yang memenuhi
1 1 1 1
+ + ··· + =
x1 + 1998 x2 + 1998 xn + 1998 1998
Buktikan √
n
x1 x2 · · · xn
≥ 1998
n−1
1.3. KETAKSAMAAN RATAAN ARITMATIK - RATAAN GEOMETRIK11

1998
Pn P
Solusi Misalkan yi = xi +1998 maka j=1 yj = 1 dan 1 − yi = j6=i yj jadi
berdasarkan AM-GM
v
n
un
X uY
1 − yi = yj ≥ (n − 1) t yj
n−1

j6=i j6=i

kalikan semua (1 − yi ) untuk i = 1, 2, · · · , n diperoleh


v
Yn u n n
uY n−1 Y
1 − yi ≥ (n − 1)n n−1 t y
i = (n − 1)n
yi
i=1 i=1 i=1

dengan mensubtitusi balik yi diperoleh


(x1 x2 · · · xn ) (n − 1)n (1998)n

(x1 + 1998)(x2 + 1998) · · · (xn + 1998) (x1 + 1998)(x2 + 1998) · · · (xn + 1998)
yang setara dengan √
n
x1 x2 · · · xn
≥ 1998.
n−1
Contoh 1.3.5 (AMM Januari 2006, Marian Tetiva) Misalkan a, b, c adalah
panjang dari sisi-sisi segitiga dengan R adalah jari-jari lingakran luar dan r
adalah jari-jari lingkaran dalam. Buktikan
2
4a2 4b2 4c2

R
≥ · ·
r 4a3 − (b − c)2 4b3 − (a − c)2 4c3 − (a − b)2
Solusi Oleh Fajar Yuliawan
Misalkan s menyatakan setengah keliling dari segitiga dan A adalah luas-nya.
Maka berlaku rumus R = abc A
4A dan r = s , jadi

R abc · s abc · s abc


= = =
2r 8A2 8s(s − a)(s − b)(s − c) 8(s − a)(s − b)(s − c)
p
dimana kita telah menggunakan rumus Hero A = s(s − a)(s − b)(s − c). Ke-
mudian kita klaim bahwa ketaksamaan berikut berlaku
2
4a2

a
≥ .
4a3 − (b − c)2
p
(s − b)(s − c)
p
Untuk membuktikan ini, misalkan x = 2 (s − b)(s − c), maka
p p p
x = (2s − 2b)(2s − 2c) = (a + c − b)(a + b − c) = a2 − (b − c)2

dan ketaksamaan yang akan kita buktikan adalah

a

4a2
 √
≥ 2 2
⇔ (3a2 + x2 )2 ≥ 16a3 x ⇔ 3a2 + x2 ≥ 4 a3 x
x 3a + x
12 BAB 1. KETAKSAMAAN RATAAN

Sedangkan dengan AM-GM kita peroleh



4

3a2 + x2 = a2 + a2 + a2 + x2 ≥ 4 a6 x2 = 4 a3 x.

Dengan cara yang serupa kita peroleh


2
4b2

b

4b3 − (a − c)2
p
(s − a)(s − c)
dan 2
4c2

c

4c − (b − a)2
3
p
(s − b)(s − a)
kalikan ketiganya kita dapatkan ketaksamaan pada soal.

1.4 Ketaksamaan Akar Rataan -AM


Ketaksamaan lain yang akan dibahas adalah sebuah ketaksamaan yang berben-
tuk mirip dengan AM. Ketaksamaan ini berhubungan
q dengan Root Mean Square
a2 +a2 +···+a2
atau disingkat RMS yang didefinisikan sebagai 1 2
n
n
. Kami akan
menyatakan ketaksamaan RMS-AM dan memberikan buktinya sebagai latihan
Teorema 1.4.1 Jika a1 , a2 , · · · , an adalah bilangan - bilangan riil , maka berlaku
r
a21 + a22 + · · · + a2n |a1 | + |a2 | + · · · + |an |

n n
dengan kesamaan terjadi jika dan hanya jika a1 = a2 = · · · = an .
Berikut ini diberikan beberapa contoh, yang pertama diambil dari olimpiade
nasional Inggris dan contoh kedua yang lebih susah diambil dari olimpiade na-
sional China.

Contoh 1.4.1 (British Mathematical Olympiad 2003) Jika x, y, dan z


adalah bilangan riil tak negatif yang memenuhi x2 + y 2 + z 2 = 1, buktikan
1
x2 yz + xy 2 z + xyz 2 ≤
3
Solusi Dengan RMS-AM kita peroleh

3(x2 + y 2 + z 2 ) ≥ (x + y + z)2 .

Dengan x2 + y 2 + z 2 = 1 diperoleh

x+y+z ≤ 3

Kemudian dengan RMS-GM diproleh


r
x2 + y 2 + z 2 √ 1
≥ 3 xyz ⇐⇒ xyz ≤ √ .
3 3 3
1.4. KETAKSAMAAN AKAR RATAAN -AM 13

Jadi
1 √ 1
x2 yz + xy 2 z + xyz 2 = xyz(x + y + z) ≤ √ 3 =
3 3 3
Contoh 1.4.2 (China Team Selection Test 1998) Misalkan n ≥ 2 adalah
bilangan bulat positif. Jika x1 , x2 , · · · , xn adalah bilangan riil sehingga
n
X n−1
X
2
xi + xi xi−1 = 1.
i=1 i=1

Diberikan sebuah bilangan bulat k dengan 1 ≤ k ≤ n, tentukan nilai maximum


dari |xk |.

Solusi Perhatikan bahwa

x1 2 + (x1 + x2 )2 + (x2 + x3 )2 + · · · + (xn−1 + xn )2 + xn 2 = 2

Dengan menggunakan RMS-AM dan ketaksamaan segitiga kita peroleh


r
x1 2 + (x1 + x2 )2 + · · · + (xk−1 + xk )2
k
|x1 | + |x1 + x2 | + · · · + |xk−1 + xk |

k
|x1 − (x1 + x2 ) + · · · + (−1)k−1 (xk−1 + xk )|

k
|xk |
=
k
Jadi kita peroleh

xk 2
x1 2 + (x1 + x2 )2 + · · · + (xk−1 + xk )2 ≥
k
Dengan cara yang serupa kita peroleh

xk 2
xn 2 + (xn−1 + xn−2 )2 + · · · + (xk+ + xk )2 ≥ .
k
Tambahkan kedua buah hasil tersebut diperoleh

2 = x1 2 + (x1 + x2 )2 + (x2 + x3 )2 + · · · + (xn−1 + xn )2 + xn 2


 
1 1
≥ + xk 2
k n−k+1
jadi
2k(n − k + 1)
|xk | ≤
n−1
Ini memberikan kita motivasi bahwa nilai maksimum dari |xk | adalah 2k(n−k+1)
n−1 ,
untuk membuktikan pernyataan ini kita cukup memperlihatkan bahwa nilai
14 BAB 1. KETAKSAMAAN RATAAN

tersebut memang dapat dicapai oleh xk yaitu dengan memperlihatkan kasus


kesamaan. Perhatikan bahwa pada penggunaan RMS-AM dan ketaksamaan
segitiga kita dapatkan bahwa kesamaan terjadi jika dan hanya jika

x1 = −(x1 + x2 ) = x1 + x3 = · · · = (−1)k−1 )(xk−1 + xk )

dan
xk + xk+1 = −(xk+1 + xk+2 ) = · · · = (−1)n−1 xn
Oleh karena itu kesamaan terjadi jika dan hanya jika
(
(−1)k−i xkk i jika i = 1, 2, · · · , k − 1,
xi =
(−1)i−k xkn−k+1
(n+1−i)
jika i = k + 1, · · · , n.

Latihan 2
1. Tunjukkan bahwa untuk bilangan positif a, b, dan c berlaku

(a + b + c)(ab + ac + bc) ≥ 9abc

2. Jika a , b , c adalah bilangan riil positif yang memenuhi abc = 1

a + b + c ≤ a2 + b2 + c2

3. Jika a , b , c adalah bilangan riil positif. Buktikan

(a + b + c)2 ≥ 3(ab + ac + bc)

4. Buktikan ketaksamaan RMS-AM !


5. Jika x, y, z > 0 buktikan
x y z
p + p + p ≤1
x+ (x + y)(x + z) y+ (y + z)(y + x) z+ (z + y)(z + x)

Hint: Gunakan cara yang mirip dengan pada soal IMO 2001, disini r = 21 .
6. Jika a1 , a2 , · · · , an adalah bilangan riil positif buktikan
 
1 1 1
(a1 + a2 + · · · + an ) + + ··· + ≥ n2
a1 a2 an

7. (Rusia) Jika x, y, z adalah bilangan riil positif yang memenuhi xyz = 1,


buktikan
(x + y)(x + z)(y + z) ≥ 4(x + y + z − 1)
8. (USAMO 1998) Misalkan a0 , a1 , · · · , an adalah bilangan pada interval
0, π2 sehingga


 π  π  π
tan a0 − + tan a1 − + · · · + tan an −
4 4 4
1.5. KETAKSAMAAN JENSEN 15

buktikan tan a0 tan a1 · · · tan an ≥ nn−1 . Hint: Langkah yang digunakan mirip
dengan solusi Soal Vietnam 1998.
9. (APMO 1998) Misalkan a, b, c adalah bilangan riil positif. Buktikan
   
 a b  c a+b+c
1+ 1+ 1+ ≥2 1+ √ 3
b c a abc
10.(Mircea Lascu) Jika a, b, c adalah bilangan riil positif dengan abc = 1,
buktikan
1 1 1 1
2 2
+ 2 2
+ 2 2
≤ .
a + 2b + 3 b + 2c + 3 c + 2a + 3 2

1.5 Ketaksamaan Jensen


Ketaksamaan yang kita kenal sebelum ini masih dapat digeneralisasi lagi, ke-
taksamaan AM-GM merupakan alat yang ampuh untuk menyelesaikan soal -
soal pertidaksamaan yang terpisah, tapi ketaksamaan yang akan diperkenalkan
berikut dapat membuat solusi -solusi yang lebih kreatif dan hasil - hasil yang
lebih umum, sebelumnya kita mulai dengan memberikan sebuah definisi.
Definisi Sebuah fungsi bernilai riil f dikatakan konveks di suatu interval I jika
dan hanya jika untuk setiap x1 , x2 ∈ I dan 0 ≤ λ ≤ 1 berlaku
λf (x1 ) + (1 − λ)f (x2 ) ≥ f (λx1 + (1 − λ)x2 )
Sedangkan fungsi f dikatakan konkav jika dan hanya jika
λf (x1 ) + (1 − λ)f (x2 ) ≤ f (λx1 + (1 − λ)x2 )
Secara visual, grafik fungsi konveks merupakan sebuah kurva yang cekung
keatas, atau jika dihubungkan dengan definisi, pada fungsi konveks apabila di-
ambil sebarang dua titik pada kurva f maka garis yang menghubungkan dua
titik tersebut pasti berada di atas kurva f itu sendiri.
Teorema 1.5.1 (Ketaksamaan Jensen) Misalkan f adalah fungsi riil yang
konveks pada interval I, jika x1 , x2 , · · · , xn ∈ I dan λi dengan i = 1, 2, · · · , n
adalah bilangan riil nonegatif yang memenuhi λ1 + λ2 + · · · + λn = 1 maka
berlaku
λ1 f (x1 ) + λ2 f (x2 ) + · · · + λn f (xn ) ≥ f (λ1 x1 + λ2 x2 + · · · + λn xn )
Bukti Untuk kasus n = 2 kita memperoleh definisi, misalkan teorema benar
sampai pada n buah variabel, jika λ1 + λ2 + · · · + λn+1 = 1 maka
n+1 n
! P
n 
X X λ i xi
xi f (xi ) ≥ λi f Pi=1
n + λn+1 f (x n+1 )
i=1 i=1 i=1 λi
n
! !
 P n 
X λ i x i
≥f λi Pi=1
n + λn+1 xn+1
i=1 i=1 λi
n+1
!
X
=f xi λ i . Q.E.D
i=1
16 BAB 1. KETAKSAMAAN RATAAN

Pada teorema diatas tidak dinyatakan kapan kesamaan dapat tercapai, tapi
dapat dilihat dengan mudah bahwa apabila x1 = x2 = · · · = xn maka kesamaan
akan tercapai. Sedangkan apabila fungsi tersebut memenuhi kriteria pada defin-
isi fungsi konveks dengan tanda ≤ diganti < maka kesamaan hanya akan terjadi
jika x1 = x2 = · · · = xn .
Untuk fungsi konkav ketaksamaan Jensen tetap berlaku dengan tanda ≥
diganti ≤. Akan tetapi sulit bagi kita untuk menentukan apakah sebuah fungsi
konvek dengan menggunakan definsi secara langsung, teorema-teorema berikut
kami nyatakan tanpa bukti karena pembuktiannya membutuhkan kalkulus yang
terlalu jauh dari bahasan kita.
Definisi Fungsi riil f dikatakan menaik di interval I jika untuk setiap x, y ∈ I
berlaku
x < y ⇒ f (x) < f (y)
dan dikatakan menurun jika berlaku
x < y ⇒ f (x) > f (y)
Teorema 1.5.2 Jika f adalah fungsi riil dan f 0 (x) > 0 untuk setiap x di I
maka f adalah menaik di I, sebaliknya jika f 0 (x) < 0 di I maka f menurun.
Teorema 1.5.3 Fungsi riil f konveks di I jika dan hanya jika f 00 (x) ≥ 0 di I,
sedangkan f konkav di I jika dan hanya jika f 00 (x) ≤ 0 di I.
Teorema 1.5.4 Jika f adalah fungsi menaik pada garis bilangan riil maka:
jika f dan g konvek di interval I, maka f ◦ g juga konveks.
jika f dan g konkav di interval I, maka f ◦ g juga konkav.

Contoh 1.5.1 (USAMO 1974) Untuk a, b, c > 0 buktikan bahwa aa bb cc ≥


(abc)(a+b+c)/3 !
Solusi Ambil f (x) = ln x, karena f 00 (x) = −1x2 ≤ 0 untuk x > 0, maka fungsi f
konkav pada interval x > 0. Jadi berdasarkan ketaksamaan Jensen
 2
a + b2 + c2

a ln a b ln b c ln c
+ + ≥ ln
a+b+c a+b+c a+b+c a+b+c
Dari RMS-AM diperoleh
a2 + b2 + c2 a2 + b2 + c2
   
a+b+c a+b+c
≥ =⇒ ln ≥ ln
a+b+c 3 a+b+c 3
Jadi  
a ln a b ln b c ln c a+b+c
+ + ≥ ln
a+b+c a+b+c a+b+c 3
Dengan AM-GM kita peroleh
√ √
 
a+b+c 3 a+b+c 3

≥ abc =⇒ ln ≥ ln abc
3 3
1.5. KETAKSAMAAN JENSEN 17

Jadi √ 
a b c 3
ln a + ln b + ln c ≥ ln abc
a+b+c a+b+c a+b+c
Dengan penyederhanaan akan diperoleh

aa bb cc ≥ (abc)(a+b+c)/3

Sering kali soal - soal yang dapat diselesaikan dengan ketaksamaan Jensen
dapat digeneralisasi menjadi banyak vaiabel,

Contoh 1.5.2 India 1995 Misalkan x1 , x2 , · · · , xn adalah bilangan riil positif


dengan hasil penjumlahannya sama dengan 1. Buktikan bahwa
r
x xn n
√ 1 + ··· + √ ≥
1 − x1 1 − xn n−1
1
Solusi Perhatikan bahwa fungsi f (x) = √1−x adalah fungsi konveks untuk
x < 1 karena f 00 (x) = 4(1−x)5/2 ≥ 0 untuk x < 1, dimana kita tahu bahwa x1 +
3

x2 + · · · xn = 1 dan xi < 1 untuk i = 1, 2, · · · , n maka berdasarkan ketaksamaan


Jensen
x xn 1
√ 1 + ··· + √ ≥p
1 − x1 1 − xn 1 − (x21 + x22 + · · · + x2n )
Sedangkan dari RMS-AM kita peroleh

(x1 + x2 + · · · + xn )2
(x21 + x22 + · · · + x2n ) ≥
n
Karena x1 + x2 + · · · xn = 1 maka
1
(x21 + x22 + · · · + x2n ) ≥ ⇐⇒ n(x21 + x22 + · · · + x2n ) ≥ 1
n
⇐⇒ n(x21 + x22 + · · · + x2n ) − n ≥ 1 − n
n 1
⇐⇒ ≥ 2 2
1−n (x1 + x2 + · · · + x2n ) − 1
n 1
⇐⇒ ≤
n−1 1 − (x21 + x22 + · · · + x2n )
r
n 1
⇐⇒ ≤p
n−1 1 − (x1 + x22 + · · · + x2n )
2

Dan kemudian dapat disimpulkan


x1 xn 1
√ + ··· + √ ≥ p
1 − x1 1 − xn 1− (x21 + x22 + · · · + x2n )
r
n

n−1
18 BAB 1. KETAKSAMAAN RATAAN

Pada subbab sebelumnya, telah diberikan sebuah bukti dari soal IMO 2001,
seringkali kita tertarik untuk melihat pendekatan atau solusi lain dari suatu
masalah. Solusi lain tersebut dapat memperkaya kreativitas berpikir kita dan
seringkali berguna apabila kita mengerjakan persoalan lain yang boleh jadi mem-
punyai ide yang serupa. Sangat dianjurkan bagi para pembaca untuk mencoba
menemukan solusi alternatif dari suatu soal. Berikut ini salah satu solusi lain
dari soal IMO 2001 dengan menggunakan ketaksamaan Jensen.

Contoh 1.5.3 (Generalisasi soal International Math Olympiad 2001)


Buktikan bahwa
a b c 3
√ +√ +√ ≥√
a2 + λbc b2 + λac c2 + λab 1+λ
untuk a, b dan c bilangan riil positif dan bilangan riil λ ≥ 8.

Solusi Fungsi f (x) = √1x konveks untuk x > 0 karena f 00 (x) = 3


4x5/2
> 0 untuk
x > 0, aplikasikan ketaksamaan Jensen pada fungsi ini dengan
a b c
λ1 = λ2 = λ1 =
a+b+c a+b+c a+b+c

x1 = a2 + λbc x2 = b2 + λac x3 = c2 + λab


maka akan didapatkan ketaksamaan
a b c a+b+c
√ +√ +√ ≥q
a2 + λbc 2
b + λac 2
c + λab a +b3 +c3 +3λabc
3
a+b+c

Jadi sekarang tinggal memeriksa apakah


a+b+c 3
q ≥√
a3 +b3 +c3 +3λabc 1+λ
a+b+c

Tapi dengan AM-GM kita peroleh

(a + b + c)3 = (a3 + b3 + c3 ) + 3(ab2 + ac2 + ba2 + bc2 + ca2 + cb2 ) + 6abc


≥ (a3 + b3 + c3 ) + 18abc + 6abc
= (a3 + b3 + c3 ) + 24abc

Dengan AM-GM kita peroleh a3 + b3 + c3 ≥ 3abc dan karena λ − 8 ≥ 0 maka

(λ + 1)(a + b + c)3 = (λ + 1)(a3 + b3 + c3 ) + 24(λ + 1)abc


= 9(a3 + b3 + c3 ) + (λ − 8)(a3 + b3 + c3 ) + 24(λ + 1)abc
≥ 9(a3 + b3 + c3 ) + 3(λ − 8)abc + 24(λ + 1)abc
= 9(a3 + b3 + c3 ) + 27λabc
= 9(a3 + b3 + c3 + 3λabc).
1.5. KETAKSAMAAN JENSEN 19

Sekarang kita mempunyai

(a + b + c)2 9
(λ + 1)(a + b + c)3 ≥ 9(a3 + b3 + c3 + 3λabc) ⇐⇒ a3 +b3 +c3 +3λabc

a+b+c
(λ + 1)

Akarkan kedua ruas maka


a+b+c 3
q ≥√
a3 +b3 +c3 +3λabc λ+1
a+b+c

seperti yang diinginkan.

Contoh 1.5.4 (AM-GM terboboti) Buktikan bahwa untuk bilangan positif


a1 , a2 , · · · , an dan bilangan positif λ1 + λ2 + · · · + λn = 1 berlaku

λ1 a1 + λ2 a2 + · · · + λn an ≥ aλ1 1 aλ2 2 · · · aλnn

dengan kesamaan terjadi jika dan hanya jika a1 = a2 = · · · = an .

Solusi Pertimbangkan fungsi f (x) = ln x, karena f 00 (x) = − x12 < 0 untuk


x > 0 maka fungsi tersebut konkav dan berdasarkan ketaksamaan Jensen,

λ1 ln x1 + λ2 ln x2 + · · · + λn ln xn ≤ ln(λ1 x1 + λ2 x2 + · · · + λn xn )

dimana λ1 , λ2 , · · · , λn adalah bilangan riil positif dengan hasil penjumlahan


sama dengan satu. Jadi kita peroleh

ln(xλ1 1 xλ2 2 · · · xλnn ) ≤ ln(λ1 x1 + λ2 x2 + · · · + λn xn )

sehingga dapat disimpulkan

xλ1 1 xλ2 2 · · · xλnn ≤ λ1 x1 + λ2 x2 + · · · + λn xn

Untuk melihat kapan kesamaan terjadi, perhatikan bahwa fungsi f memenuhi


f 00 (x) > 0 jadi kesamaan hanya terjadi jika x1 = x2 = · · · = xn .

Hasil diatas adalah generalisasi dari AM-GM, perhatikan bahwa jika kita
memilih λ1 = λ2 = · · · = λn = n1 kita peroleh AM-GM yang biasa. Denga
Ketaksamaan Jensen juga dapat diperoleh RMS-AM dan GM-HM terboboti.

Contoh 1.5.5 (International Mathematics Olympiad 2006) Tentukan bi-


langan riil M terkecil sehingga ketaksamaan

|ab(a2 − b2 ) + bc(b2 − c2 ) + ca(c2 − a2 )| ≤ M (a2 + b2 + c2 )2

berlaku untuk sembarang bilangan riil a , b dan c


20 BAB 1. KETAKSAMAAN RATAAN

Solusi Solusi resmi


Pertama - tama kita akan tunjukkan bahwa

|ab(a2 − b2 ) + bc(b2 − c2 ) + ca(c2 − a2 )| = |(a − b)(b − c)(a − c)(a + b + c)|

Pertimbangkan

P (t) = bt(t2 − b2 ) + bc(b2 − c2 ) + ct(c2 − t2 )

Perhatikan P (t) adalah polinomial derajat 3 dalam t dan juga P (b) = P (c) =
P (−b − c) = 0 dab juga P (t) = 0 jika b = c. Jadi

P (t) = (b − c)(t − b)(t − c)(t + b + c)

sehingga diperloleh

|ab(a2 − b2 ) + bc(b2 − c2 ) + ca(c2 − a2 )| = |P (a)| = |(b − c)(a − b)(a − c)(a + b + c)|

Sekarang masalahnya ekuivalen dengan mencari M terkecil sehingga

|(b − c)(a − b)(a − c)(a + b + c)| ≤ M (a2 + b2 + c2 )2

Cukup bagi kita untuk hanya mempertimbangkan kasus dimana a ≥ b ≥ c,


karena untuk kasus lain contohnya a ≥ c ≥ b kita dapat menuliskan

|(b − c)(a − b)(a − c)(a + b + c)| = |(c − b)(a − b)(a − c)(a + b + c)|

yang berarti tidak masalah jika posisi b digantikan oleh c, dan jika b ≥ a ≥ c
kita dapat menuliskan

|(a − c)(b − a)(b − c)(a + b + c)| = |(b − c)(a − b)(a − c)(a + b + c)|

Kasus - kasus lain ditangani serupa, dimana posisi a , b dan c dapat saling
dipertukarkan tanpa mengubah persoalan awal. Jadi cukup bagi kita untuk
membuktikan kasus a ≥ b ≥ c. Oleh kerena itu (a − b) ≥ 0 dan (b − c) ≥ 0 lalu
dengan AM-GM kita dapatkan
2
(a − c)2

(a − b) + (b − c)
|(a − b)(b − c)| = (a − b)(b − c) ≤ =
2 4

dengan kesamaan terjadi jika dan hanya jika a − b = b − c atau a + c = 2b.


Kemudian dengan RMS-AM kita peroleh
2
(a − c)2 (a − b)2 + (b − c)2
  
(a − b) + (b − c)
= ≤
4 2 2

Jadi
3(a − c)2 ≤ 2((a − b)2 + (b − c)2 + (a − c)2 )
1.5. KETAKSAMAAN JENSEN 21

Dengan kesamaan juga terjadi jika dan hanya jika a + c = 2b. Dari hasil - hasil
diatas kita dapatkan

|(b − c)(a − b)(a − c)(a + b + c)|


1
≤ |(a − c)3 (a + b + c)|
4
1p
= (a − c)6 (a + b + c)2
4s
3
(a − b)2 + (b − c)2 + (a − c)2

1
≤ 8 (a + b + c)2
4 3
2

s
3
2  4 (a − b)2 + (b − c)2 + (a − c)2

= (a + b + c)2 
2 3

Dengan AM-GM terboboti kita peroleh

|(b − c)(a − b)(a − c)(a + b + c)|


2

s
 2 2 2
3
2  4 (a − b) + (b − c) + (a − c)
≤ (a + b + c)2 
2 3
√ 3/4 !2
(a − b)2 + (b − c)2 + (a − c)2

2 2 1/4
= ((a + b + c) )
2 3
√   2
2 3 (a − b)2 + (b − c)2 + (a − c)2

1 2
≤ + (a + b + c)
2 4 3 4

9 2 2
= (a + b2 + c2 )
32

Kita lihat bahwa M = 9322 memenuhi. Untuk melihat bahwa M tersebut adalah
yang terkecil, kita harus tunjukkan bahwa tanda samadengan dapat terjadi,
yaitu terdapat a, b dan c sehingga

9 2 2
|(b − c)(a − b)(a − c)(a + b + c)| = (a + b2 + c2 )
32
Untuk menyelidiki ini, kita lihat beberapa ketakamaan yang telah kita gunakan
untuk mendapatkan nilai M yaitu AM-GM dua kali dan AM-GM terboboti.
Pada penggunaan 2 kali AM-GM kita dapatkan bahwa kesamaan terjadi jika
dan hanya jika a+c = 2b, dan dari penggunaan AM-GM terboboti kita dapatkan
2 2
+(a−c)2
bahwa kesamaan terjadi jika dan hanya jika (a−b) +(b−c)
3 = (a2 + b2 +
2 2
c ) , jadi kesamaan terjadi jika dan hanya jika

a+c (a − b)2 + (b − c)2 + (a − c)2


b= dan = (a2 + b2 + c2 )2
2 3
22 BAB 1. KETAKSAMAAN RATAAN

a+c (a−b)2 +(b−c)2 +(a−c)2


Subtitusikan b = 2 ke persamaan 3 = (a2 + b2 + c2 )2
diperoleh
2(c − a)2 = 9(a + c)2
Jadi kondisi kesamaan adalah
a+c
b= (c − a)2 = 18b2
2
√ √
Pada saat b = 1 kita peroleh a = 1 − 23 2 dan b = 1 + 32 2 merupakan
salah satu nilai a, b dan c yang memenuhi. Nilai b yang lain dapat meng-
hasilkan nilai a √dan c yang√berbeda pula. Jadi kesamaan terjadi pada nilai
(a, b, c) = 1 − 23 2, 1, 1 + 23 2 dan pasangan terurut yang proposional. Juga


perhatikan bahwa kita telah mengasumsikan a ≥ b ≥ c karena √ perubahan


√  po-
sisi a, b dan c dapat dipertukarkan, jadi (a, b, c) = 1 − 32 2, 1 + 32 2, 1 juga
dapat menyebabkan kesamaan. Sehingga kita simpulkan kesamaan √ terjadi
√  jika
dan hanya jika nilai a, b, c adalah proposional dengan 1 − 32 2, 1, 1 + 23 2 dan
permutasinya.

Latihan 3
1. Buktikan ketaksamaan Power Mean - Arithmatic Mean yaitu untuk bilangan
riil positif a1 , a2 , · · · , an dan p > 1 atau p < 0 berlaku
p
ap1 + ap2 + · · · + apn

a1 + a2 + · · · + an

n n
dengan kesamaan terjadi jika dan hanya jika a1 = a2 = · · · = an .
2. Jika α, β dan γ adalah sudut - sudut dari segitiga lancip. Buktikan
3
cos α + cos β + cos γ ≤ .
2
3. Misalkan pada sebuah lingkaran diambil n buah titik, kemudian dari tiap -
tiap titik tersebut ditarik garis ke pusat lingkaran sehingga diperoleh poligon
didalam lingkaran tersebut. Buktikan bahwa poligon dengan luas terbesar diper-
oleh jika semua titik yang bersebelahan mempunyai jarak sama (atau dengan
kata lain apabila poligon yang dimaksud adalah poligon teratur).
4. (Korea 1998). Jika a, b, c adalah bilangan riil memenuhi a + b + c = abc
maka buktikan
1 1 1 3
√ +√ +√ ≤
1+a 2 1+b 2 1+c 2 2
Dan tentukan kapan terjadi kesamaan. √ √
Peringatan: Fungsi f (x) = √1+x 1
2
tidak konveks pada − 22 < x < 22 . Lihat
soal 2.
5. Jika x, y, z adalah bilangan riil positif dengan kondisi x+y +z = 1 , buktikan
r
x y z 3
√ +√ +√ ≥
1−x 1−y 1−z 2
1.5. KETAKSAMAAN JENSEN 23

6.(Germany 2005) Misalkan a, b, c adalah bilangan riil positif dengan ab +


ac + bc = 1, buktikan
r r r
3 1 3 1 3 1 1
+ 6b + + 6c + + 6a ≤
a b c abc
Dan tentukan kapan kasus samadengan terjadi.
7. Untuk x ≥ y ≥ 1, buktikan
x y 1 y x 1
√ +√ +√ ≥√ +√ +√ .
x+y y+1 x+1 x+y x+1 y+1

Hint: Misalkan x = y + a dan y = 1 + b dengan a, b ≥ 0.


24 BAB 1. KETAKSAMAAN RATAAN
Bab 2

Ketaksamaan Norm

2.1 Ketaksamaan Cauchy-Schwarz (CS), Minkowski


dan Hölder
Teorema 2.1.1 Jika diberikan sebuah pasangan terurut u = (a1 , a2 , · · · , an )
dan v = (b1 , b2 , · · · , bn ) adalah bilangan riil, maka berlaku

u  v ≤ |u||v|

atau ditulis
n
!2 n
! n
!
X X X
ak bk ≤ ak bk
k=1 k=1 k=1

dengan kesamaan terjadi jika ai = c bi untuk suatu konstan c.

n
X
Bukti Kita mulai dari (ak x + bk )2 ≥ 0, oleh karena itu kita dapatkan
k=0

n
X
(a2k x2 + 2ak bk x + b2k )
k=0

we define
n
X n
X n
X
A= a2k B= b2k C= ak bk
k=0 k=0 k=0

n
X
kemudian (ak x + bk )2 = Ax2 + 2Bx + C ≥ 0, jelas bahwa A ≥ 0, . Jika
k=0
A = 0 , maka semua ai sama dengan 0 jadi C = 0 dan B = 0 sehingga tidak

25
26 BAB 2. KETAKSAMAAN NORM

ada yang perlu dibuktikan. Sekarang misalkan A > 0 maka


 
B C
Ax2 + 2Bx + C = A x2 + 2 x +
A A
 2  2 !
2 B B B C
= A x +2 x+ − +
A A A A
2 !
AC − B 2

B
= A x+ +
A A2
AC − B 2
 
≥ A 0+
A2
AC − B 2
=
A

2
Jadi AC−B
A adalah nilai minimum dari Ax2 + 2Bx + C yang tercapai pada saat
−B
x = A . Karena kita ketahui bahwa Ax2 + 2Bx + C ≥ 0 untuk setiap x, maka
AC − B 2
pada saat x = −B
A kita peroleh ≥ 0 atau AC ≥ B 2 . Subtitusi balik
A
A, B, dan C didapatkan

n
!2 n
! n
!
X X X
ak bk ≤ a2k b2k
k=1 k=0 k=0

seperti yang diinginkan.

Perlu ditekankan lagi bahwa ketaksamaan CS berlaku untuk semua bilangan


riil, ditambah lagi kita mempunyai dua buah variabel yang memiliki indeks
yaitu ai dan bi , kepiawaian kita dalam menentukan ai dan bi akan membuat
ketaksamaan ini menjadi sangat berguna.

Contoh 2.1.1 Jika a, b, c adalah bilangan - bilangan riil maka buktikan abc(a+
b + c) ≤ (ab)2 + (ac)2 + (bc)2

Solusi Dengan
p menggunakan
p ketaksamaan Cauchy-Schwarz kita dapatkan xy+
yz+zx ≤ x2 + y 2 + z 2 y 2 + z 2 + x2 = x2 +y 2 +z 2 . Jadi kita dapatkan untuk
sebarang bilangan riil x, y, z

xy + yz + xz ≤ x2 + y 2 + z 2 .

Karena x, y, z adalah bilangan riil sebarang, maka kita boleh memisalkan x = ab,
y = ac dan z = bc. Jadi

abc(a + b + c) = (ab)(ac) + (ac)(bc) + (ab)(bc) ≤ (ab)2 + (ac)2 + (bc)2 .


2.1. KETAKSAMAAN CAUCHY-SCHWARZ (CS), MINKOWSKI DAN HÖLDER 27

Misalkan
√ pada
√ ketaksamaan
√ Cauchy- Schwarz kita menggunakan pasangan
terurut ( b1 , b2 , · · · , bn ) dan ( √ab1 , √ab2 , · · · , √abn ) dimana ak adalah bilan-
1 2 n
gan riil dan bk > 0 maka diperoleh
n p
!2 n
! n !
X ak X X a2
k
bk · √ ≤ bk
bk bk
k=1 k=1 k=1

Yang ekuivalen dengan


a21 a2 a2 a1 + a2 + · · · + an
+ 2 + ··· + n ≥ .
b1 b2 bn b1 + b2 + · · · + bn
√  
Dengan kesamaan terjadi jika bk = c √abk ⇐⇒ bk = c ak , untuk suatu
k
konstanta c. Perhatikan baik - baik hasil yang baru kita dapatkan tersbut,
ketaksaman tersebut akan sering berguna, terutama untuk mereduksi bentuk
pecahan terpisah menjadi pecahan tunggal. Biasanya bentuk ini disebut ketak-
samaan CS bentuk Engel. Kita ilustrasikan penggunaannya pada pembuktian
salah satu ketaksamaan populer berikut.

Contoh 2.1.2 (Ketaksamaan Nesbitt) Buktikan untuk bilangan riil positif


a , b dan c,
a b c 3
+ + ≥
b+c a+c a+b 2
Dengan kesamaan terjadi jika dan hanya jika a = b = c.

Bukti Perhatikan bahwa


a b c a2 b2 c2
+ + = + +
b+c a+c a+b ab + ac ab + bc ac + bc
(a + b + c)2

(ab + ac) + (ab + bc) + (ac + bc)
Dimana kita telah mereduksi tiga bentuk pecahan menjadi satu bentuk pecahan.
Jadi cukup bagi kita untuk membuktikan
(a + b + c)2 3

(ab + ac) + (ab + bc) + (ac + bc) 2
yang ekuivalen dengan

(a + b + c)2 ≥ 3(ab + ac + bc)

Sedangkan dari ketaksamaan CS diperoleh

ab + ac + bc ≤ a2 + b2 + c2 ⇐⇒ 3(ab + ac + bc) ≤ (a + b + c)2

Dengan kesamaan terjadi jika a2 = k(ab+ac), b2 = k(bc+ab) dan c2 = k(ac+bc).


Yaitu jika a = k(b + c), b = k(b + c) dan c = k(b + c), kita simpulkan kesamaan
terjadi untuk a = b = c.
28 BAB 2. KETAKSAMAAN NORM

Contoh 2.1.3 (IMO Short List 1990) Jika ab + bc + cd + ad = 1 dengan


a, b, c, d > 0 maka buktikan

a3 b3 c3 d3 1
+ + + ≥
b+c+d c+d+a d+a+b a+b+c 3

Solusi Pertama - tama perhatikan bahwa

a3 b3 c3 d3
+ + +
b+c+d c+d+a d+a+b a+b+c
a4 b4 c4 d4
= + + +
ab + ac + ad bc + bd + ba cd + ca + cb da + db + dc
Dengan menggunakan CS-Engel diperoleh

a4 b4 c4 d4
+ + +
ab + ac + ad bc + bd + ba cd + ca + cb da + db + dc
(a2 + b2 + c2 + d2 )2

2(ab + ac + cd + ad + bc + bd)

Melalui ketaksamaan AM-GM dan CS


p p
a2 + b2 + c2 + d2 ≥ 2 a2 + b2 c2 + d2
≥ 2(ac + bd).

Jadi sekarang kita dapatkan

(a2 + b2 + c2 + d2 )2 (a2 + b2 + c2 + d2 )2 (a2 + b2 + c2 + d2 )2


= ≥ .
2(ab + ac + cd + ad + bc + bd) 2 + 2(ac + bd) 2 + a2 + b2 + c2 + d2

Sedangkan dengan CS diperoleh a2 + b2 + c2 + d2 ≥ ab + bc + cd + ad = 1.


Misalkan X = a2 + b2 + c2 + d2 , maka (X − 1) ≥ 0, dan jelas bahwa 3X + 2 ≥ 0.
Jadi 3X 2 − X − 2 = (3X + 2)(X − 1) ≥ 0. Maka

X2 1
3X 2 − X − 2 ≥ 0 ⇐⇒ ≥
2+X 3
Dengan kata lain kita telah menunjukkan

a3 b3 c3 d3 (a2 + b2 + c2 + d2 )2
+ + + ≥
b+c+d c+d+a d+a+b a+b+c 2 + a2 + b2 + c2 + d2
X2
=
2+X
1
≥ .
3
Seperti yang diinginkan.
2.1. KETAKSAMAAN CAUCHY-SCHWARZ (CS), MINKOWSKI DAN HÖLDER 29

Kita telah melihat bahwa CS bentuk Engel telah membuat bebearapa soal
menjadi mudah, bahkan beberapa diantaranya adalah soal - soal dari IMO short-
list, anda dapat mencoba beberapa soal pada problem set di bab ini. Tapi perlu
diketahui bahwa tidak semua ketaksamaan bentuk pecahan dapat diselesaikan
dengan cara serupa, ada beberapa diantaranya yang terlalu ‘kuat’ untuk dise-
lesaikan dengan CS.
Contoh 2.1.4 (Math Magazine 1634) Jika a , b dan c adalah bilangan riil
positif, tentukan bilangan riil k terkecil sehingga
ab ac bc
+ + ≤ k(a + b + c)
a + b + 2c b + c + 2b b + c + 2a
dan tentukan kapan terjadi kasus samadengan.
Solusi Klaim: jika a, b, c > 0 maka
ab ac bc 1
+ + ≤ (a + b + c)
a + b + 2c b + c + 2b b + c + 2a 4
yang berarti k = 41 akan memenuhi ketaksamaan, kemudian akan ditunjukkan
bahwa k tersebut adalah yang terkecil dengan memperlihatkan adanya kesamaan.
Kita akan membuktikan klaim ini, perhatikan bahwa dengan CS engel kita per-
oleh
ab ab 4ab
+ ≥
a+c b+c a + b + 2c
bc bc 4bc
+ ≥
b+a c+a b + c + 2a
ac ac 4ac
+ ≥
c+b b+a c + a + 2b

Tambahkan ketiga ketaksamaan diatas kita peroleh


 
ab ac bc
4 + +
a + b + 2c b + c + 2b b + c + 2a
ab ab bc bc ac ac
≤ + + + + +
a+c b+c b+a c+a c+b b+a
=a+b+c
yang setara dengan ketaksamaan pada soal sehingga klaim terbukti sekarang
akan diperiksa apakah k = 14 itu adalah yang terkecil, jelas bahwa jika a = b = c
maka kita memperoleh kesamaan sehingga k tersebut memang yang terkecil.
Mari kita tinjau lebih lanjut kasus kesamaan, karena kita menggunakan CS
sebanyak tiga kali maka tanda samadengan berlaku jika dan hanya jika
a + c = λ1 = (b + c) a + b = λ2 = (a + c) b + c = λ3 = (a + b)
untuk suatu bilangan riil positif λ1 , λ2 dan λ3 . Jadi tanda samadengan berlaku
jika dan hanya jika a = b = c.
30 BAB 2. KETAKSAMAAN NORM

Contoh 2.1.5 (Moldova 2006) Jika a, b, c > 0 dengan abc = 1 buktikan

a+3 b+3 c+3


+ + ≥3
(a + 1)2 (b + 1)2 (c + 1)2

Solusi Kita akan kembali menggunakan Lemma yang sama seperti pada soal
China 2005, tapi disini lemma akan dibuktikan dengan cara berbeda untuk
bilangan riil positif.
Lemma: Jika x, y adalah bilangan riil positif maka
1 1 1
2
+ 2

(x + 1) (y + 1) 1 + xy

Bukti Lemma:
Dengan menggunakan CS diperoleh
√ √ √
( x + x · xy)2 ≤ (x + y)(1 + xy)

yang ekuivalen dengan


  
1 x 1

(1 + x)2 x+y 1 + xy

dengan cara yang serupa


  
1 y 1

(1 + y)2 x+y 1 + xy

apabila keduanya ditambahkan


  
1 1 x+y 1
+ ≥
(1 + x)2 (1 + y)2 x+y 1 + xy

merupakan lemma yang diminta.


Kemudian dengan menerapkan Lemma diperoleh
2 2 2 1 1 1
+ + ≥ + +
(a + 1)2 (b + 1)2 (c + 1)2 1 + ab 1 + bc 1 + ac
a+1 1 b+1 1
Dengan menambahkan kedua ruas dengan (a+1)2 = a+1 , (b+1)2 = b+1 dan
c+1 1
(c+1)2 = c+1 diperoleh

a+3 b+3 c+3 1 1 1 1 1 1


2
+ 2
+ 2
≥ + + + + +
(a + 1) (b + 1) (c + 1) 1 + ab 1 + bc 1 + ac 1 + a 1 + b 1 + c

sedangkan kondisi abc = 1 menyebabkan bagian sebelah kanan menjadi


a b c 1 1 1
+ + + + + =3
1+a 1+b 1+c 1+a 1+b 1+c
2.1. KETAKSAMAAN CAUCHY-SCHWARZ (CS), MINKOWSKI DAN HÖLDER 31

Contoh 2.1.6 (Vasile Cirtoaje) Untuk bilangan riil positif a, b, c, d. Buk-


tikan
a−b b−c c−d d−a
+ + + ≥0
b+c c+d a+d a+b
Solusi Melalui aplikasi dari CS-Engel

a−b b−c c−d d−a


+ + +
b+c c+d a+d a+b
a−b−c+c b−c−d+d c−d−a+a d−a−b+b
= + + +
b+c c+d a+d a+b
a+c b+d c+a d+b
= + + + −4
b+c c+d a+d a+b
   
1 1 1 1
= (a + c) + + (b + d) + −4
b+c a+d c+d a+d
   
4 4
≥ (a + c) + (b + d) −4
a+b+c+d a+b+c+d
= 0

Teorema berikut merupakan salah satu generalisasi dari Ketaksamaan Cauchy-


Schwarz.

Teorema 2.1.2 (Ketaksamaan Hölder) Jika λ1 , λ2 , · · · , λn adalah bilangan


riil positif yang memenuhi λ1 + λ2 + · · · + λn = 1, maka untuk sebarang bilangan
riil positif aij berlaku
 λi !
n
Y Xm m
X n
Y
λi
 aij  ≥ aij .
i=1 j=1 j=1 i=1

Bukti Misalkan  λi


Xm
Ai =  aij 
j=1

maka berdasarkan AM-GM terboboti kita peroleh


! ! !
a1j a2j anj aλ1j1 aλ2j2 aλnjn
λ1 1/λ
+ λ2 1/λ
+ · · · + λ1 ≥ ···
(A1 ) 1 (A2 ) 2 (An )1/λn A1 A2 An
Pm
Kemudian dengan melakukan operator j=1 pada kedua ruas lalu mensubti-
1/λ Pm
tusikan Ai i = j=1 aij kita peroleh

Pm Pm Pm  
m
a a a
  X
j=1 1j j=1 2j j=1 nj 1
λ1 Pm +λ2 Pm +· · ·+λn Pm ≥  aij a2j · · · anj 
j=1 a1j j=1 a2j j=1 anj A1 A2 · · · An j=1
32 BAB 2. KETAKSAMAAN NORM

Sehingga
 !
  m n
1 X Y
1 = λ1 + λ2 + · · · + λn ≥ Qn  aij 
i=1 Ai j=1 i=1

yang setara dengan


 λi !
n
Y m
X m
X n
Y
 aij  ≥ aλiji .
i=1 j=1 j=1 i=1

Contoh 2.1.7 Buktikan bahwa untuk x, y, z adalah bilangan riil positif maka

9 4
(x2 y 2 z 2 + xyz + 1)2 ≤ (x + 1)(y 4 + 1)(z 4 + 1)
8
Solusi oleh ThAzNi (Mathlinker)
Pertama - tama kita buktikan dulu Lemma berikut
Lemma: Jika t adalah bilangan riil positif maka berlaku

9 4
(t + 1)3 ≥ (t6 + t3 + 1)2
8
bukti lemma Perhatikan bahwa

9(t4 +1)3 −8(t6 +t3 +1)2 = (t−1)4 (t8 +4t7 +10t6 +4t5 −2t4 +4t3 +10t2 +4t+1)

karena dengan AM-GM berlaku t8 + 1 ≥ 2t4 maka

9(t4 + 1)3 − 8(t6 + t3 + 1)2 ≥ 0

dan lemma telah terbukti. Dengan lemma tersebut diperoleh

9(x4 + 1)3 − 8(x6 + x3 + 1)2 ≥ 0 9(y 4 + 1)3 − 8(y 6 + y 3 + 1)2 ≥ 0

9(z 4 + 1)3 − 8(z 6 + z 3 + 1)2 ≥ 0.


Hasil perkaliannya

9 4
(x + 1)(y 4 + 1)(z 4 + 1) ≥ [(x6 + x3 + 1)(y 6 + y 3 + 1)(z 6 + z 3 + 1)]2/3
8
Dengan ketaksamaan Hölder diperoleh

[(x6 + x3 + 1)1/3 (y 6 + y 3 + 1)1/3 (z 6 + z 3 + 1)1/3 ]2 ≥ (x2 y 2 z 2 + xyz + 1)2

Ketaksamaan selanjutnya merupakan salah satu generalisasi dari ketaksamaan


Cauchy-Schwarz yang biasa disebut Ketaksamaan Minkowski.
2.1. KETAKSAMAAN CAUCHY-SCHWARZ (CS), MINKOWSKI DAN HÖLDER 33

Teorema 2.1.3 (Minkowski) Misalkan r > s adalah bilangan riil yang tidak
sama dengan 0, maka untuk bilangan positif aij berlaku
 !s/r 1/s
  r/s 1/r
m
X n
X Xn Xm
arij ≥ asij  
    
j=1 i=1 i=1 j=1

Latihan 4
1. (IMO Shortlist 1993) Jika a, b, c, d adalah bilangan riil positif, buktikan
a b c d 2
+ + + ≥
b + 2c + 3d c + 2d + 3a d + 2a + 3b a + 2b + 3c 3
2. (Romania 1997) Jika a, b, c adalah bilangan riil positif, buktikan

a2 b2 c2 bc ac ab
+ 2 + 2 ≥1≥ 2 + 2 + 2
a2 + 2bc b + 2ac c + 2ab a + 2bc c + 2ab c + 2ab
3. (Canada 2002) Jika a, b, c adalah bilangan riil positif, buktikan

a3 b3 c3
+ + ≥a+b+c
bc ac ab
4. (Ireland 1998) Buktikan bahwa jika a, b, c adalah bilangan riil positif maka
 
9 1 1 1
≤2 + +
a+b+c a+b b+c a+c
dan    
1 1 1 1 1 1 1
+ + ≤ + +
a+b b+c a+c 2 a b c
1 1 1
5. (Iran 1998) Misalkan x, y, z > 1 dan x + y + z = 2. Buktikan
√ √ p √
x+y+z ≥ x−1+ y−1+ z−1

6. Jika x, y, z bilangan riil positif, buktikan


 √ s   
√ √ 2 xy 3 x+y x+y+z
3(x + xy + 3
xyz) ≤ 8 + x
x+y 2 3

7.(IMO 1995) Jika a, b, c adalah bilangan riil positif yang memenuhi abc = 1,
buktikan
1 1 1 3
+ + ≥
a3 (b + c) b3 (a + c) c3 (b + a) 2
8.(USAMO 1997) Jika x, y, z adalah bilangan riil positif maka buktikan
1 1 1 1
+ + ≤
a3 + b3 + abc b3 + c3 + abc c3 + a3 + abc abc
34 BAB 2. KETAKSAMAAN NORM

Hint: Kalikan kedua ruas dengan abc lalu lanjutkan seperti solusi IMO 2001.
9.(China,????) Misalkan ri , si , ti , ui , vi adalah bilangan riil tidak kurang dari
1, untuk i = 1, 2, · · · , n dan misalkan R, S, T, U, V, masing-masing merupakan
rataan aritmatik (Aritmatic Mean) dari ri , si , ti , ui , vi secara berurutan. Buk-
tikan
n  n
Y ri si ti ui vi + 1 RST U V + 1

r s t u v −1
i=1 i i i i i
RST U V − 1

10. (IMO 2005) Misalkan x, y, z adalah bilangan riil positif sehingga xyz ≥ 1.
Buktikan
x5 − x2 y5 − y2 z5 − z2
5 2 2
+ 2 5 2
+ 2 ≥0
x +y +z x +y +y x + y2 + z5
x2 +y 2 +z 2 yz+y 2 +z 2
Hint: Buktikan x5 +y 2 +z 2 ≤ x2 +y 2 +z 2 .

2.2 Rearrangement & Ketaksamaan Chebishev


Sekarang
P merupakan P saat yang tepat untuk memperkenalkan pengunaan no-
tasi cyc dan sym dalam menuliskan ekspresi matematik. Notasi P ini sangat
membantu kita dalam menulis solusi. Kita mulai dengan notasi sym . Mis-
alkan E(a, b, c) adalah fungsi tiga variabel, didefinsikan
X
E(a, b, c) = E(a, b, c)+E(a, c, b)+E(b, a, c)+E(b, c, a)+E(c, a, b)+E(c, b, a)
sym

P
Dengan kata lain sym E(a, b, c) merupakan penjumlahan fungsi E(a, b, c) di-
ikuti oleh permutasi dari a , b dan c pada fungsi yang bersangkutan. Kita dapat
mengeneralisasikan notasi ini untuk fungsi n variabel E(x1 , x2 , · · · , xn ). Untuk
kasus 3 variabel diatas terdapat 6 = 3! suku (permutasi), sedangkan untuk 4
variabel terdapatP4! suku dan seterusnya n! suku untuk n varibel.
Untuk notasi cyc didefinisikan sebagai
X
E(a, b, c) = E(a, b, c) + E(b, c, a) + E(c, a, b)
cyc

P
Dengan kata lain cyc merupakan setengah dari penjumlahan siklik dimana
posisi a, b dan c berganti menurut a menjadi b , b menjadi c dan c menjadi
a. Untuk kasus 3 variabel terdapat 21 (3!) suku. Generalisasi untuk n variabel
dapat dikembangkan secara serupa.

Contoh 2.2.1 Jika kita ingin menulis xy + yz + xz maka dapat ditulis


X X1
xy atau xy
cyc sym
2
2.2. REARRANGEMENT & KETAKSAMAAN CHEBISHEV 35

Contoh 2.2.2 Ketaksamaan Nesbitt pada contoh 1.5.2 dapat ditulis


X a 3 X a
≥ atau ≥3
cyc
b+c 2 sym
b+c

Contoh 2.2.3 Ekspansi dari (x + y + z)2 (xy + yz + xz) dapat dituliskan


X X X
x3 y + 5 x2 yz + x2 y 2
sym cyc sym

Seringkali didalam menulis solusi kita menggunakan frase ” Tanpa kehilan-


gan keumuman ” atau dalam bahwa inggris di singkat WLOG yaitu ”Without
Loss of Generality”, dikarenakan untuk menyelesaikan suatu soal kita cukup
mempertimbangkan sedikit kasus. Perhatikan bahwa sering kali berguna bagi
kita didalam menyelesaikan soal pertidaksamaan berbentuk simetris atau siklis
dengan mengasumsikan a ≥ b ≥ c atau x1 ≥ x2 ≥ · · · ≥ xn (Ingat, penalaran
ini telah kita gunakan sebelumnya pada solusi soal IMO 2006). Misalkan pada
ketaksamaan Nesbitt diperbolehkan WLOG a ≥ b ≥ c karena jika
a b c
f (a, b, c) = + +
b+c a+c a+b
dapat dilihat bahwa tidak masalah mana yang terbesar diantara a, b atau c,
nilai f tetap sama, dapat dilihat f (1, 2, 3) = f (1, 3, 2) = f (2, 1, 3) = f (2, 3, 1) =
f (3, 1, 2) = f (3, 2, 1) = 17
10 . Faktanya semua ketaksamaan yang simetris berlaku
seperti ini, sedangkan pada ketaksamaan siklis, pertama - tama kita lakukan
pembuktian dengan syarat a ≥ b ≥ c kemudian lakukan juga pembuktian den-
gan syarat a ≥ c ≥ b. Hal ini tentunya sangat berguna, karena untuk mem-
buktikan suatu ketaksaman yang simetrik kita cukup membuktikan satu kasus
saja daripada menuliskan bukti kasus per kasus yang pada dasarnya sebenarnya
mempunyai jalan yang sama.
Contoh 2.2.4 Adalah suatu keputusan yang tidak tepat bagi kita untuk mengam-
bil langkah WLOG a ≥ b ≥ c dalam membuktikan ketaksamaan
X
a2 b = a2 b + b2 c + c2 a ≥ 3
cyc

dengan syarat abc = 1 dan a , b , c bilangan riil positif.


 Karena jika f (a, b, c) =
a2 b + b2 c + c2 a maka f 1, 21 , 2 = 5 6= 17
4 = f 1, 2, 1
2 .

P Berikutnya
P akan diberikan contoh untuk menunjukkan bagaimana notasi
cyc dan sym dapat membantu dalam penulisan solusi
Sesuatu yang akan kita pelajari pada bahasan selanjutnya sedikit berbeda
dengan yang sebelumnya, Rearrangement sebenarnya bukanlah benar - benar
ketaksamaan seperti halnya AM-GM, CS atau ketaksamaan Jensen, ini lebih
kepada metode yang digunakan untuk menyelesaikan sebuah persoalan ketak-
samaan. Sedangkan dari Rearrangement dapat diturunkan sebuah ketaksamaan
yang sering disebut ketaksamaan Chebishev. Untuk memulai bahasan kita ten-
tang Rearrangement, mari kita tinjau contoh sederhana berikut.
36 BAB 2. KETAKSAMAAN NORM

Contoh 2.2.5 Sebuah mangkok berisi 24 keping uang logam yang terdiri dari 8
Keping uang 1000 , 8 Keping uang 500 dan 8 keping uang 100. Seseorang boleh
melakukan pengambilan uang tersebut dengan cuma - cuma tapi dengan syarat:
- Pengambilan dilakukan tiga kali.
- Pada pengambilan pertama hanya boleh diambil 4 keping.
- Pada pengambilan kedua hanya boleh diambil 2 keping.
- Pada pengambilan ketiga hanya boleh diambil 6 keping.
Bagaimana cara pengambilan agar diperoleh
a. Uang maksimum?
b. Uang minimum?

Solusi a. Tentu saja untuk mendapatkan banyak uang strateginya adalah den-
gan mengambil uang yang bernilai besar yaitu 1000 sebanyak mungkin, dan
kemudian diikuti yang kedua terbesar. Jadi pada pengambilan pertama kita
ambil uang bernilai 500 yaitu berdasarkan syarat 4 keping, pada pengambilan
kedua ambil 2 keping uang 100 dan pada pengambilan yang ketiga adalah 6
keping uang 1000. Jadi kita dapatkan

4(500) + 2(100) + 6(1000) = 8200

Pengambilan dengan cara lain tapi tetap menurut syarat diatas akan membuat
uang yang dihasilkan lebih kecil dari 8200.
b. Untuk mendapatkan uang minimum dapat diperoleh dengan cara

4(500) + 2(1000) + 6(100) = 4600

Jadi bagaimanapun cara pengambilan (dengan syarat diatas), uang yang diper-
oleh pasti lebih dari 4600.

Sekarang kita akan menyatakan Rearrangement yang diambil dari contoh


diatas.
Teorema 2.2.1 Misalkan terdapat dua pasangan terurut (a1 , a2 , · · · , an ) dan
(b1 , b2 , · · · , bn ) dengan ai dan bi adalah bilangan riil untuk i = 1, 2, · · · , n. Jika
a1 ≥ a2 ≥ · · · ≥ an dan b1 ≥ b2 ≥ · · · ≥ bn , maka hasil kali titik dengan nilai
maksimum adalah
SM = a1 b1 + a2 b2 + · · · + an bn
dan hasil kali titik dengan nilai minimum adalah

Sm = a1 bn + a2 bn−1 + · · · + an b1

Dengan kata lain jika pasangan terurut (a1 , a2 , · · · , an ) dikalikan titik dengan
permutasi dari pasangan terurut (b1 , b2 , · · · bn ) katakanlah (bσ(1) , bσ2 , · · · , bσn )
maka berlaku
Sm ≤ Sp ≤ SM .
Dengan tanda sama dengan terjadi jika dan hanya jika a1 = a2 = · · · = an atau
b1 = b2 = · · · = bn
2.2. REARRANGEMENT & KETAKSAMAAN CHEBISHEV 37

Bukti Tanpa kehilangan keumuman, cukup bagi kita untuk mempertimbangkan


apabila pengurutan pada ekspresi Sp diperoleh dengan menukar dua buah bi-
langan riil ai dan aj . Misalkan ai ≥ aj dan bi ≥ bj , dan

SM = a1 b1 + a2 b2 + · · · + ai bi + · · · + aj bj + · · · + an bn

Sp = a1 b1 + a2 b2 + · · · + aj bi + · · · + ai bj + · · · + an bn
maka Sp − SM = bi (aj − ai ) + bj (ai − aj ) = (bi − bj )(aj − ai ) ≤ 0, jadi Sp ≤ SM .
Untuk melihat kapan kesamaan terjadi, perhatikan bahwa (bi − bj )(aj − ai ) = 0
jika dan hanya jika ai = aj atau bi = bj . Dengan cara yang sama dapat
dibuktikan Sm ≤ Sp .

Contoh 2.2.6 (International Mathematical Olympiad 1975) Diberikan bi-


langan riil x1 ≤ x2 ≤ · · · ≤ xn dan y1 ≤ y2 ≤ · · · ≤ yn . Jika (z1 , z2 , · · · , zn )
adalah permutasi dari (y1 , y2 , · · · , yn ). Buktikan bahwa
n
X n
X
(xi − yi )2 ≤ (xi − zi )2
i=1 i=1

Solusi Karena (z1 , z2 , · · · , zn ) adalah permutasi dari (y1 , y2 , · · · , yn ) maka


n
X n
X
x2i = zi2
i=1 i=1

Dengan demikian
n
X n
X n
X n
X
(xi − yi )2 ≤ (xi − zi )2 ⇐⇒ xi yi ≥ xi zi
i=1 i=1 i=1 i=1

yang dapat diperoleh dari rearrangement.

Contoh 2.2.7 (International Mathematical Olympiad 1983) Misalkan a


, b dan c adalah panjang sisi - sisi sebuah segitiga. Buktikan bahwa

a2 b(a − b) + b2 c(b − c) + c2 a(c − a) ≥ 0

Solusi Perhatikan bahwa ketaksamaan diatas siklik, jadi kita cukup membuk-
tikan untuk kasus dimana a ≥ b ≥ c dan a ≥ c ≥ b. Karena a, b, dan c adalah
panjang sisi pada segitiga maka berlaku

a+b>c b+c>a a+c>b

Misalkan a ≥ b ≥ c maka b(c + a − b) − a(b + c − a) = (a − b)(a + b − c) ≥ 0


sehingga b(c + a − b) ≥ a(b + c − a). Dengan cara yang sama kita dapatkan
c(a + b − c) ≥ b(c + a − b), jadi

c(a + b − c) ≥ b(c + a − b) ≥ a(b + c − a)


38 BAB 2. KETAKSAMAAN NORM

Juga perhatikan bahwa karena a ≥ b ≥ c maka


1 1 1
≥ ≥
c b a
1 1 1

Jadi berdasarkan Rearrangement pada c, b, a dan (c(a + b − c), b(c + a −
b), a(b + c − a)) kita perloleh
1 1 1
c(a + b − c) + b(c + a − b) + a(b + c − a) = a + b + c
c b a
adalah ekspresi yang paling maksimum, dan juga ekspresi lain yaitu
1 1 1
c(a + b − c) + b(c + a − b) + a(b + c − a)
b a c
harus memenuhi
1 1 1
c(a + b − c) + b(c + a − b) + a(b + c − a) ≤ a + b + c
b a c
kalikan kedua ruas dengan abc diperoleh
ac2 (a + b − c) + b2 c(c + a − b) + a2 b(b + c − a) ≤ abc(a + b + c)
yang disederhanakan mencjadi
ac2 (a − c) + b2 c(a − b) + a2 (b − a) ≤ 0 ⇐⇒ a2 b(a − b) + b2 c(b − c) + c2 a(c − a) ≥ 0
Untuk kasus a ≥ c ≥ b dengan cara yang serupa kita dapatkan
b(c + a − b) ≥ c(a + b − c) ≥ a(b + c − a)
selanjutnya kita dapat menggunakan cara yang serupa dengan kasus pertama.
Misalkan terdapat pasangan terurut (a1 , a2 , · · · , an ) dan (b1 , b2 , · · · , bn ) den-
gan a1 ≥ a2 ≥ · · · ≥ an dan b1 ≥ b2 ≥ · · · ≥ bn dan misalkan
(x1 , x2 , · · · , xn ) (xn , x1 , · · · , xn−1 ) ··· (x2 , x3 , · · · , x1 )
adalah n buah permutasi siklis berbeda dari (b1 , b2 , · · · , bn ), maka berdasarkan
Rearrangement berlaku
(a1 b1 + a2 b2 + · · · + an bn ) ≥ a1 x1 + a2 x2 + · · · + an xn
(a1 b1 + a2 b2 + · · · + an bn ) ≥ a1 xn + a2 x1 + · · · + an xn−1
······ ······
(a1 b1 + a2 b2 + · · · + an bn ) ≥ a1 x2 + a2 x3 + · · · + an x1

Jumlahkan semua ketaksamaan diatas kita peroleh


n(a1 b1 + a2 b2 + · · · + an bn ) ≥ a1 (x1 + xn + · · · + x2 ) + a2 (x2 + x1 + · · · + x3 )
+ · · · + an (xn + xn−1 + · · · + x1 )
= (a1 + a2 + · · · + an )(x1 + x2 + · · · + xn )
2.2. REARRANGEMENT & KETAKSAMAAN CHEBISHEV 39

Karena x1 + x2 + · · · + xn = b1 + b2 + · · · + bn maka

(a1 + a2 + · · · + an )(b1 + b2 + · · · + bn )
(a1 b1 + a2 b2 + · · · + an bn ) ≥
n
Dengan tanda ≥ berubah menjadi ≤ pada kasus b1 ≤ b2 ≤ · · · ≤ bn , kita telah
membuktikan

Teorema 2.2.2 Jika (a1 , a2 , · · · , an ) dan (b1 , b2 , · · · , bn ) adalah pasangan teru-


rut dari bilangan riil. Jika a1 ≥ a2 ≥ · · · ≥ an dan b1 ≥ b2 ≥ · · · ≥ bn maka

(a1 + a2 + · · · + an )(b1 + b2 + · · · + bn )
(a1 b1 + a2 b2 + · · · + an bn ) ≥
n
Jika a1 ≥ a2 ≥ · · · ≥ an dan b1 ≤ b2 ≤ · · · ≤ bn maka

(a1 + a2 + · · · + an )(b1 + b2 + · · · + bn )
(a1 b1 + a2 b2 + · · · + an bn ) ≤
n
Contoh 2.2.8 (Romania 2005, Cezar Lupu) Buktikan bahwa untuk sebarang
bilangan riil positif a , b dan c berlaku
 
b+c a+c a+b 1 1 1
+ 2 + 2 ≥2 + +
a2 b c a b c

Ketaksamaan diatas simetrik, WLOG a ≥ b ≥ c maka

1 1 1 b+c a+c a+b


≤ ≤ ≤ ≤
a b c a b c
Dengan ketaksamaan Chebisev diperoleh
  
b+c a+c a+b 1 b+c a+c a+b 1 1 1
+ 2 + 2 ≥ + + + +
a2 b c 3 a b c a b c

Dan dengan CS engel dan ketaksamaan (a+b+c)2 ≥ 3(ab+ac+bc) kita peroleh

(b + c)2 (a + c)2 (a + b)2 2(a + b + c)2


+ + ≥ ≥6
a(b + c) b(a + c) c(a + b) ab + ac + bc

jadi  
1 b+c a+c a+b
+ + ≥2
3 a b c
Dengan demikian
  
b+c a+c a+b 1 b+c a+c a+b 1 1 1
+ 2 + 2 ≥ + + + +
a2 b c 3 a b c a b c
 
1 1 1
≥ 2 + + .
a b c
40 BAB 2. KETAKSAMAAN NORM

Pn
Contoh 2.2.9 (Fajar Yuliawan ) Jika xi > 0, xi ∈ R dan i=1 xi = 1
buktikan
Xn n
X
xi 2 + nn−1 xi n (1 − xi ) ≥ 1
i=1 i=1

Solusi Kita akan membuktikan generalisasi soal tersebut yaitu


n
X n
X
xi 2 + nm−1 xi m (1 − xi ) ≥ 1
i=1 i=1
Pn
dengan i=1 xi = 1 dan m = 0, 1, 2, · · · . Soal tersebut akan dibuktikan meng-
gunakan dua lemma
Pn berikut:
Lemma 1 Jika i=1 xi = 1 dan xi ≥ 0 dengan x1 ≥ x2 ≥ · · · ≥ xn maka

1 + nn−1 x1 n − nn−1 x1 n+1 ≥ 1 + nn−1 x2 n − nn−1 x2 n+1


≥ ···
≥ 1 + nn−1 xn n − nn−1 xn n+1

Bukti Lemma 1: Kita akan membuktikan

xi n − xi n+1 ≥ xi+1 n − xi+1 n+1 i = 1, 2, · · · , n − 1

Karena xi ≥ xi+1 maka xi n−1 − xi+1 n−1 ≥ 0 dan xi n − xi+1 n ≥ 0, karena


P n
i=1 xi = 1 dan xi ≥ 0 maka 1 − (xi + xi+1 ) ≥ 0, jadi

(xi n − xi+1 n ) ≥ (xi + xi+1 )(xi n − xi+1 n )


= xi n+1 − xi+1 n+1 + xi xi+1 (xi n−1 − xn−1
i+1 )
≥ xi n+1 − xi+1 n+1

Jadi xi n − xi+1 n ≥ xi n+1 − xi+1 n+1 yang setara dengan

xi n − xi n+1 ≥ xi+1 n − xi+1 n+1

Kalikan dengan nn−1 kemudian tambahkan dengan 1 kita dapatkan

1 + nn−1 xi n − nn−1 xi n+1 ≥ 1 + nn−1 xi+1 n − nn−1 xi+1 n+1


Pn
Lemma 2 Jika xi ≥ 0 dengan i=1 xi = 1 maka untuk setiap bilangan bulat
positif m > 2 berlaku
n
X n
X n
X n
X
xi 2 + nm−1 xi m (1 − xi ) ≥ xi 2 + nm−3 xi m−2 (1 − xi )
i=1 i=1 i=1 i=1

Bukti Lemma 2: Pertama - tama tulis


n
X n
X n
X
2 m−1 m
xi + n xi (1 − xi ) = xi 2 (1 + nm−1 xi m−2 − nm−1 xi m−1 )
i=1 i=1 i=1
2.2. REARRANGEMENT & KETAKSAMAAN CHEBISHEV 41

WLOG x1 ≥ x2 ≥ · · · ≥ xn maka x1 2 ≥ x2 2 ≥ · · · ≥ xn 2 dan berdasarkan


Lemma 1
1 + nn−1 x1 n − nn−1 x1 n+1 ≥ 1 + nn−1 x2 n − nn−1 x2 n+1
≥ ···
≥ 1 + nn−1 xn n − nn−1 xn n+1
Pn
Jadi
Pberdasarkan ketaksamaan Chebishev dan ketaksamaan RMS-AM i=1 xi 2 ≥
1 n 1
n ( i=1 xi ) = n diperoleh
n
X n
X
xi 2 + nm−1 xi m (1 − xi )
i=1 i=1
n
X
= xi 2 (1 + nm−1 xi m−2 − nm−1 xi m−1 )
i=1
n
!" n
!#
1 X X
≥ xi 2 n + nm−1 xi m−2 − xi m−1
n i=1 i=1
n
" n
! n
!#
X X X
2 m−2 2 m−2 m−1
= xi + n xi xi − xi
i=1 i=1 i=1
n
" n
!#
X X
≥ xi 2 + nm−3 xi m−2 − xi m−1
i=1 i=1
n
X n
X
= xi 2 + nm−3 xi m−2 (1 − xi )
i=1 i=1

sehingga Lemma 2 terbukti. Sekarang kita akan membuktikan soal tersebut,


definsikan fungsi
n
X n
X
f (m) = xi 2 + nm−1 xi m (1 − xi ) m = 0, 1, 2, · · ·
i=1 i=1

Kita akan membuktikan yang lebih umum yaitu f (m) ≥ 1 untuk m = 0, 1, 2, · · · .


Perhatikan bahwa
n n
X
2 n−1 X 1
f (0) = xi + ≥1 karena xi 2 ≥
i=1
n i=1
n

f (1) = 1 ≥ 1
Jika m genap maka berdasarkan Lemma 2
f (m) ≥ f (m − 2) ≥ · · · ≥ f (0) ≥ 1
Jika m ganjil maka berdasarkan lemma 2
f (m) ≥ f (m − 2) ≥ · · · ≥ f (1) ≥ 1
Subtitusikan m = n maka f (n) ≥ 1 dan soal terbukti.
42 BAB 2. KETAKSAMAAN NORM

Contoh 2.2.10 (Michael Rozenberg) Jika a, b dan c adalah bilangan riil


positif. Buktikan

a2 + 2bc b2 + 2ac c2 + 2ab 3


+ + ≥ (a + b + c)
b+c a+c a+b 2
Solusi WLOG a ≥ b ≥ c maka
1 1 1
≥ ≥
2(b + c) 2(a + c) 2(a + b)
dan juga

a≥b ⇔ a(2a − 7c) ≥ b(2b − 7c)


⇔ 2a2 + 4bc − 3ac ≥ 2b2 + 4ac − 3bc
⇔ 2a2 + 4bc − 3ac − 3ab ≥ 2b2 + 4ac − 3bc − 3ab

dengan cara yang serupa 2b2 + 4ac − 3bc − 3ab ≥ 2c2 + 4ab − 3ac − 3bc. Jadi

2a2 + 4bc − 3ac − 3ab ≥ 2b2 + 4ac − 3bc − 3ab ≥ 2c2 + 4ab − 3ac − 3bc

sehingga dengan Rearrangement kita dapatkan

2a2 + 4bc − 3ac − 3ab 2b2 + 4ac − 3bc − 3ab 2c2 + 4ab − 3ac − 3bc
+ + ≥
2(b + c) 2(a + c) 2(a + b)
 
1 2 1 1 1
(a + b2 + c2 − ab − ac − bc) + + .
3 b+c a+c a+b

Karena a2 + b2 + c2 ≥ ab + ac + bc maka

2a2 + 4bc − 3ac − 3ab 2b2 + 4ac − 3bc − 3ab 2c2 + 4ab − 3ac − 3bc
+ + ≥0
2(b + c) 2(a + c) 2(a + b)
sedangkan ketaksamaan ini setara dengan

2(a2 + bc) 3
   2   2 
2(b + ac) 3 2(c + ab) 3
− a + − b + − c ≥0
2(b + c) 2 2(a + c) 2 2(a + b) 2

sehingga seperti yang diinginkan kita peroleh

a2 + 2bc b2 + 2ac c2 + 2ab 3


+ + ≥ (a + b + c).
b+c a+c a+b 2
Contoh 2.2.11 (International Mathematical Olympiad 2005) Misalkan
x, y, z adalah bilangan riil positif sehingga xyz ≥ 1. Buktikan

x5 − x2 y5 − y2 z5 − z2
+ 2 + 2 ≥0
x5 2
+y +z 2 x +y +z5 2 x + y2 + z5
2.2. REARRANGEMENT & KETAKSAMAAN CHEBISHEV 43

Solusi Kita mulai dengan membuktikan kedua buah lemma.


Lemma 1: Untuk x, y, z bilangan riil positif dengan xyz ≥ 1 berlaku
x5 y5 z5
+ 2 + 2 ≥1
x5 2
+y +z 2 5
x +y +z 2 x + y2 + z5
Bukti Lemma 1: Dengan menggunakan CS Engel kita peroleh
x6 y6 z6
+ +
x6 + xy 2 + xz 2 yx2 + y 6 + xz 2 x2 z + y 2 z + z 6
3 3 3 2
(x + y + z )

x6 + y 6 + z 6 + x(y 2 + z 2 ) + y(x2 + z 2 ) + z(x2 + y 2 )
Jadi cukup diperiksa apakah
2(x3 y 3 + z 3 x3 + y 3 z 3 ) ≥ x(y 2 + z 2 ) + y(x2 + z 2 ) + z(x2 + y 2 )
dengan kondisi xyz ≥ 1 kita tinggal membuktikan
 2 2
z 2 x2 y2 z2

x y
2 + + ≥ x(y 2 + z 2 ) + y(x2 + z 2 ) + z(x2 + y 2 )
x y z
Untuk membuktikan ini WLOG x ≥ y ≥ z maka
xy xz yz
xy ≥ xz ≥ yz dan ≥ ≥
z y x
maka berdasarkan Rearrangement
x2 y 2 z 2 x2 y2 z2  xy   
xz  yz 
+ + ≥ xz + yz + xy
x y z z y x
= x2 y + z 2 x + y 2 z

x2 y 2 z 2 x2 y2 z2  xy   
xz  yz 
+ + ≥ yz + xy + xz
x y z z y x
= y 2 x + x2 z + z 2 y
menambahkan dua ketaksamaan diatas akan melengkapi bukti lemma 1.
Lemma 2: Untuk bilangan riil x, y dan z dengan xyz ≥ 1 berlaku
x2 y2 z2
+ + ≤1
x5 + y 2 + z 2 x2 + y 5 + z 2 x2 + y 2 + z 5
dengan menggunakan ketaksamaan Cauchy dan kondisi xyz ≥ 1 kita peroleh
 2
2 2 2 2 5/2 1 2 2
(x + y + z ) = x √ +x +y
x
 
5 2 2 1 2 2
≤ (x + y + z ) +y +z
x
≤ (x5 + y 2 + z 2 ) yz + y 2 + z 2

44 BAB 2. KETAKSAMAAN NORM

jadi
x2 x2 (yz + y 2 + z 2 )
≤ .
x5 2
+y +z 2 (x2 + y 2 + z 2 )2
dengan cara yang serupa diperoleh

y2 y 2 (xz + x2 + z 2 ) z2 z 2 (xy + x2 + y 2 )
≤ dan ≤
y 5 + x2 + z 2 (x2 + y 2 + z 2 )2 x2 + y 2 + z 5 (x2 + y 2 + z 2 )2

tambahkan semua ketaksamaan ini diperoleh

x2 y2 z2
+ +
x5 + y 2 + z 2 x2 + y 5 + z 2 x2 + y 2 + z 5
2(x y + x z + y x ) + x yz + xy 2 z + xyz 2
2 2 2 2 2 2 2

(x2 + y 2 + z 2 )2
2(x2 y 2 + x2 z 2 + y 2 x2 ) + x2 yz + xy 2 z + xyz 2
=
x4 + y 4 + z 4 + 2(x2 y 2 + x2 z 2 + y 2 x2 )

Jadi tinggal dibuktikan

2(x2 y 2 + x2 z 2 + y 2 x2 ) + x2 yz + xy 2 z + xyz 2
≤1
x4 + y 4 + z 4 + 2(x2 y 2 + x2 z 2 + y 2 x2 )

yang setara dengan

x2 yz + xy 2 z + xyz 2 ≤ x4 + y 4 + z 4

ketaksamaan ini dapat dibuktikan dengan langkah berikut

x4 + y 4 + z 4 ≥ x2 y 2 + x2 z 2 + y 2 z 2
 2 2
x y + x2 z 2
  2 2
y z + x2 z 2
  2 2
y z + x2 y 2

= + +
2 2 2
≥ x2 yz + xy 2 z + xyz 2

sehingga Lemma 2 Terbukti.


Kurangkan ketaksamaan pada lemma 1 dengan ketaksamaan pada lemma 2 kita
telah menyelesaikan soal IMO ini.

Latihan 5
1. Jika a, b dan c adalah panjang sisi - sisi pada segitiga tidak degenerate,
buktikan
1 1 1 1 1 1
+ + ≥ + +
a+b−c b+c−a a+c−b a b c
2. Jika x1 , x2 , · · · , xn adalah bilangan riil positif dengan x1 x2 · · · xn = 1 dan
bilangan riil s dan t sehingga s ≥ t buktikan

x1 s + x2 s + · · · + xn s ≥ x1 t + x2 t+ · · · + xn t
2.2. REARRANGEMENT & KETAKSAMAAN CHEBISHEV 45

3.(Vasile Cirtoaje) Misalkan a, b, c adalah bilangan riil positif, buktikan


√ √ √ √
a2 + bc b2 + ac c2 + ab 3 2
+ + ≥
b+c a+c a+b 2
4.(IMO 1978) Misalkan a1 , a2 , · · · , an adalah bilangan riil yang berbeda, buk-
tikan
n n
X ai X 1
2

i=1
i i=1
i

5.(IMO 1964) Misalkan a, b, dan c adalah panjang sisi - sisi sebuah segitiga,
buktikan bahwa

a2 (b + c − a) + b2 (a + c − b) + c2 (a + b − c) ≤ 3abc

6.(Germany 2005) Untuk bilangan riil positif a, b, c dengan a + b + c = 1


buktikan  
1+a 1+b 1+c b c a
+ + ≤2 + +
1−a 1−b 1−c a b c
46 BAB 2. KETAKSAMAAN NORM
Bab 3

Penjumlahan Simetrik dan


Siklis

3.1 Normalisasi dan Subtitusi


Beberapa materi yang telah kita pelajari sebelumnya memang berhasil menyele-
saikan banyak persoalan pertidaksamaan, materi - materi tersebut dapat dikatakan
materi standar dan masih belum cukup untuk menyelesaikan soal - soal yang
tingkat kesulitannya tinggi, apalagi seiring berjalannya waktu soal - soal per-
tidaksamaan tidak menjadi lebih mudah, pembaca mungkin menyadari bahwa
banyak soal olimpiade dari mancanegara yang mudah dalam selang waktu 90an.
Pertama - tama kita akan belajar tentang normalisasi. Perhatikan kembali
Ketaksamaan Nesbitt, yang menyatakan:
Untuk a, b, c > 0
a b c 3
+ + ≥ .
b+c a+c a+b 2
Membuktikan ketaksamaan ini sama saja dengan membuktikan ketaksamaan:
Jika a, b, c > 0 dengan a + b + c = 1
a b c 3
+ + ≥
b+c a+c a+b 2
karena jika a + b + c = k dan subtitusi a = kx, b = kx dan c = kx memberikan
kx ky kz 3
+ + ≥
ky + kz kx + kz kx + ky 2
dengan kx + ky + kz = k. Dengan kata lain untuk membuktikan ketaksamaan
Nesbitt dapat dengan cara membuktikan
x y z 3
+ + ≥
y+z x+z x+y 2
dengan kondisi x + y + z = 1.

47
48 BAB 3. PENJUMLAHAN SIMETRIK DAN SIKLIS

Perubahan variabel (a, b, c) menjadi (x, y, z) tidak lebih dari perubahan


nama variabe saja (kita katakan disini dummy variable), jadi tidak masalah
jika kita tetap menuliskan dalam variabel (a, b, c) yaitu :
Untuk a, b, c > 0 dengan a + b + c = 1 maka
a b c 3
+ + ≥
b+c a+c a+b 2
Secara tidak langsung kita telah menyatakan ”WLOG a + b + c = 1”.
Suatu hal yang memungkinkan hal ini pada ketaksamaan nesbitt adalah
karena pada ketaksamaan tersebut subtitusi a = kx, b = ky, c = kz berujung
pada hilangnya k. Ketaksamaan yang seperti ini kita katakan homogen. Setiap
ketaksamaan dapat kita tulis ke bentuk f (x1 , x2 , · · · , xn ) ≥ 0, misalkan pada
ketaksamaan Nessbit kita dapat menuliskan
a b c 3
f (a, b, c) = + + −
b+c a+c a+b 2
dan f (a, b, c) ≥ 0. Ketaksamaan tersebut dikatakan homogen jika

f (λx1 , λx2 , · · · , λxn ) = f (x1 , x2 , · · · , xn )

untuk sebarang bilangan riil λ.


Tambahan lagi tidak diharuskan bagi kita untuk mengasumsikan a + b +
c = 1, dengan leluasa pada ketaksamaan homogen kita boleh mengasumsikan
ab + ac + bc = 1, a2 + b2 + c2 = 1, abc = 1 atau lebih banyak lagi. Terlebih
lagi konstanta yang dipilih tidak harus angka 1, kita boleh mengasumsikan
a + b + c = r untuk nilai r yang tergantung pada soal. Misalkan jika a, b, c > 0
dan ketaksamaannya homogen, kita dapat mengasumsikan a + b + c = 3, tapi
tentu saja tidak diperbolehkan untuk mengasumsikan a + b + c = −1.
Setelah mengasumsikan sebuah syarat, maka kita akan mendapatkan sebuah
ketaksamaan yang tidak homogen lagi. Kita rangkum apa yang telah dijelaskan
diatas pada contoh berikut
Contoh 3.1.1 (Ketaksamaan Nesbitt) Jika a, b, c > 0 maka

a b c 3
+ + ≥
b+c a+c a+b 2
Solusi Karena ketaksamaan tersebut homogen maka WLOG a + b + c = 1
dengan 0 < a, b, c < 1. Sehingga ketaksamaan diatas dapat ditulis dalam bentuk
a b c 3
+ + ≥
1−a 1−b 1−c 2
Untuk membuktikan ketaksamaan diatas kita gunakan Lemma berikut
Lemma: Jika x adalah bilangan riil dengan x < 1 maka berlaku
x 9x − 1

1−x 4
3.1. NORMALISASI DAN SUBTITUSI 49

Bukti Lemma:
Ketaksamaan akan diperoleh melalui rutunan langkah berikut

(3x − 1)2 ≥ 0 ⇔ 9x2 + 4x + 1 ≥ 10x


⇔ 4x ≥ −9x2 + 10x − 1
⇔ 4x ≥ (9x − 1)(1 − x)
x 9x − 1
⇔ ≥
1−x 4

sehingga Lemma terbukti.


Dengan mengaplikasikan lemma tersebut kita dapatkan
a b c 9a − 1 9b − 1 9c − 1
+ + ≥ + +
1−a 1−b 1−c 4 4 4
9(a + b + c) − 3
=
4
3
= .
2
Contoh 3.1.2 (USAMO 2003) Misalkan a, b, c adalah bilangan riil positif,
buktikan
(2a + b + c)2 (a + 2b + c)2 (a + b + 2c)2
+ + ≤8
2a2 + (b + c)2 2b2 + (a + c)2 2c2 + (a + b)2

Solusi Karena ketaksamaan tersebut homogen maka WLOG a + b + c = 3, jadi


ketaksamaan diatas berubah menjadi

(a + 3)2 (b + 3)2 (c + 3)2


+ + ≤ 8.
2a2 + (3 − a)2 2b2 + (3 − b)2 2c2 + (3 − c)2

Kita bentuk Lemma berikut


Lemma: Jika x adalah bilangan riil dengan x ≥ − 43 maka berlaku

(x + 3)2 4
≤ (x + 1)
2x2 + (3 − x)2 3

Bukti Lemma:
Ketaksamaan akan diperoleh melalui rutunan langkah berikut

(x − 1)2 (4x + 3) ≥ 0 ⇔ 4x3 − 5x2 − 2x + 3 ≥ 0


⇔ 4x3 − 4x2 + 4x + 12 ≥ x2 + 6x + 9
⇔ 4(x + 1)(x2 − 2x + 3) ≥ x2 + 6x + 9
4 x2 + 6x + 9
⇔ (x + 1) ≥
3 3(x2 − 2x + 3)
4 (x + 3)2
⇔ (x + 1) ≥ 2
3 2x + (3 − x)2
50 BAB 3. PENJUMLAHAN SIMETRIK DAN SIKLIS

membuktikan Lemma.
Dengan menggunakan Lemma kita dapatkan
(a + 3)2 (b + 3)2 (c + 3)2 4
+ 2 + 2 ≤ (a + b + c + 3)
2a2+ (3 − a)2 2b + (3 − b)2 2c + (3 − c)2 3
= 8.
Sesuatu yang tampak tidak natural pada dua bukti diatas terletak pada
Lemma yang digunakan. Pada contoh 1.8.1 kita menggunakan
x 9x − 1

1−x 4
pada contoh 1.8.2 kita menggunakan
(x + 3)2 4
2 2
≤ (x + 1)
2x + (3 − x) 3
bagaimana kedua Lemma ini bisa terpikirkan? Untuk kasus pada contoh ini
terdapat penjelasan matematis dari pertanyaan ini. Kedua Lemma tersebut
diturunkan dari teorema berikut:
Teorema 3.1.1 Misalkan f adalah sebuah fungsi konveks pada interval buka I,
jika x0 ∈ I maka berlaku
f (x) ≥ f (x0 ) + f 0 (x0 )(x − x0 )
Pada fungsi konkav tanda ≥ diganti dengan ≤. Walupun tidak terlalu sulit,
bukti formal dari teorema ini memerlukan kalkulus dan tidak akan diberikan
disini. Ide yang mendasari pembentukan Teorema 1.8.1 adalah fakta bahwa
pada fungsi konveks garis singgung pada suatu titik (x0 , f (x0 )) selalu berada
dibawah grafik fungsi tersebut. Sedangkan dari kalkulus kita ketahui bahwa
persamaan garis singgung pada titik (x0 , f (x0 )) adalah y = f (x0 ) + f 0 (x0 )(x −
x0 ).
Berkutnya akan diperlihatkan penurunan lemma pada contoh 1.8.1 dengan
teorema ini. Pada contoh 1.8.1 setelah diberikan syarat tanda samadengan
terjadi jika a = b = c = 31 , jadi agar teorema bisa dipakai dengan mengawetkan
kasus samadengan kita pilih x0 = 31 pada teorema dengan fungsi f (x) = 1−x x
.
Fungsi f konveks di x ∈ (0, 1) maka berdasarkan teorema 1.8.1
    
0 1 1 1
f (x) ≥ f x− +f
3 3 3
yang setara dengan
x 9x − 1

1−x 4
2
Sedangkan pada contoh 1.8.2 digunakan fungsi konkav f (x) = x2(x+3)
+(3−x)2 , dengan
kasus samadengan terjadi jika a = b = c = 1. Jadi kita gunakan teorema dengan
x0 = 1 sehingga
f (x) ≤ f 0 (1)(x − 1) + f (1)
3.1. NORMALISASI DAN SUBTITUSI 51

yang setara dengan


(x + 3)2 4
≤ (x + 1)
2x2 + (3 − x)2 3
Teorema 1.8.1 seringkali hanya digunakan untuk memperoleh lemma, sedan-
gkan pembuktiannya dilakukan dengan cara aljabar biasa seperti pemfaktoran,
pengkuadratan, dll. Perhatikan bahwa teorema 1.8.1 dapat gagal menghasilkan
lemma jika f tidak konveks atau tidak konkav. Bahkan pada contoh berikut
walaupun f konkav tapi teorema tersebut tidak membantu menyelesaikan soal.

Contoh 3.1.3 Jika x, y, z > 0 buktikan


x y z
p + p + p ≤1
x+ (x + y)(x + z) y+ (y + z)(y + x) z + (z + y)(z + x)

Solusi Sisi kiri ketaksamaan diatas ekuivalen dengan


x y z
p + p + p
x+ x2 + xz + yx + yz y+ y2 + xy + zy + xz z+ +z 2 + yz + yz + xz

Karena ketaksamaan homogen maka WLOG xy + yz + xz = 1 jadi kita peroleh


x y z
√ + p + √ ≤1
x + x + 1 y + y + 1 z + +z 2 + 1
2 2

Untuk membuktikan ini kita lakukan subtitusi a = tan α , b = tan β dan c =


tan γ dengan 0 < α, β, γ < π2 maka kita harus membuktikan:
Jika tan α tan β + tan α tan γ + tan β tan γ = 1 berlaku

sin α sin β sin γ


+ + ≤1
1 + sin α 1 + sin β 1 + sin γ
π
Lemma jika 0 < α, β, γ < 2 dan

tan α tan β + tan α tan γ + tan β tan γ = 1

maka
π
α+β+γ =
.
2
Bukti Lemma Pertimbangkan segitiga lancip ABC misalkan γ = ∠ABC di-
mana kita peroleh

AC BC
tan ABC = dan tan BAC =
BC AC
π
karena BAC = 2 − ABC maka kita peroleh identitas
π  1
tan −γ =
2 tan γ
52 BAB 3. PENJUMLAHAN SIMETRIK DAN SIKLIS

1
Kemudian kalikan kedua ruas dengan tan α+tan β kita dapatkan
π  tan α + tan β
tan −γ =
2 tan α tan γ + tan β tan γ
tan α + tan β
=
1 − tan αβ
= tan(α + β)
karena semua α, β, γ ada diinterval 0, π2 maka kita simpulkan π2 − γ = α + β


dan lemma terbukti.


x
Berikutnya karena fungsi f (x) = 1+x konkav menaik, dan fungsi g(x) = sin x
konkav pada interval [0, π] maka berdasarkan teorema 1.5.4 h(x) = (f ◦ g)(x) =
sin x
1+sin x konkav pada [0, π]. Jadi berdasarkan ketaksamaan Jensen dan Lemma

sin α sin β sin γ 3 sin π6


+ + ≤ =1
1 + sin α 1 + sin β 1 + sin γ 1 + sin π6
Pada contoh diatas walaupun fungsi φ(x) = x+√xx2 +1 juga konkav pada in-
terval [0, ∞) namun apabila kita menggunakan ketaksamaan Jensen pada fungsi
ini kita akan mendapatkan
x y z x+y+z
√ + p + √ ≤ q
x + x + 1 y + y 2 + 1 z + +z 2 + 1
2 x+y+z
+ x+y+z

+1
3 3

sehingga selanjutnya kita harus membuktikan


x+y+z
q ≤1
x+y+z x+y+z 2

3 + 3 + 1
dibawah kondisi xy + yz + xz √ = 1. Tapi ketaksamaan ini salah karena jika
x = y = 13 dan z = 34 maka 5 ≤ 13 yang jelas salah.
Kemudian jika kita menggunakan teorema 1.8.1 maka pembuktian akan
berlanjut dengan lemma berikut

t 3 1
√ − t− ≤0
t + t2 + 1 6 6
Walaupun lemma ini benar, tapi apabila kita gunakan selanjutnya akan dihara-
pkan √
3 1
(x + y + z) + ≤ 1
6 2
Sekali lagi ini salah karena jika x = y = 31 dan z = 43 maka dari ketaksamaan

ini akan diperoleh 3 ≤ 1 yang jelas salah.
Hal ini berarti subtitusi trigonometri yang dilakukan pada solusi contoh 1.8.3
adalah sangat esensial (walaupun pada subbab 2 terdapat solusi yang lebih mu-
dah). Pada soal - soal pertidaksamaan, teknik subtitusi sering dilakukan seperti
pada Latihan 3 no 4. Berikut ini adalah soal - soal yang dapat diselesaikan den-
gan teknik subtitusi.
3.1. NORMALISASI DAN SUBTITUSI 53

Contoh 3.1.4 (IMO 1995) Misalkan a, b, c adalah bilangan riil positif dengan
abc = 1 buktikan
1 1 1 3
+ 3 + 3 ≥
a3 (b + c) b (a + c) c (a + b) 2

Solusi Ketaksamaan diatas tidak homogen tapi dengan melakukan subtitusi


a = x1 , b = y1 dan c = z1 maka xyz = 1 dan kita peroleh ketaksamaan

x2 y2 z2 3
+ + ≥
y+z z+x x+y 2
x y z
Ketaksamaan simetrik jadi WLOG x ≥ y ≥ z maka y+z ≥ x+z ≥ x+y dan
berdasarkan ketaksamaan Chebishev
x2 y2 z2
  
x+y+z x y z
+ + ≥ + +
y+z z+x x+y 3 y+z z+x x+y

Karena xyz = 1 maka berdasarkan AM-GM x+y+z 3 ≥ 1. Lalu ketaksamaan


x y z
Nesbitt menyatakan y+z + z+x + x+y ≥ 32 . Perkalian keduanya akan melengkapi
solusi.

Contoh 3.1.5 (AMM Januari 2006, Oleg Faynshteyn) Misalkan x, y dan


z adalah bilangan riil positif yang memenuhi x2 + y 2 + z 2 = 1 dan misalkan n
adalah bilangan bulat positif. Tunjukkan bahwa

x y z (2n + 1)1+1/2n
+ + ≥
1 − x2n 1 − y 2n 1 − z 2n 2n
Solusi Oleh Hendrata Dermawan
Misalkan
√ √ √
a = x( 2n 2n + 1) b = y( 2n 2n + 1) c = z( 2n 2n + 1)

Karena 0 < x, y, z < 1 maka 0 < a2n , b2n , c2n < 2n√ + 1. Sedangkan kondisi
x2 + y 2 + z 2 = 1 berubah menjadi a2 + b2 + c2 = n 2n + 1 dan ketaksamaan
yang akan dibuktikan menjadi
√n
a b c 2n + 1
2n
+ 2n
+ 2n

2n + 1 − a 2n + 1 − b 2n + 1 − c 2n

dengan subtitusi a2 + b2 + c2 = n 2n + 1, ketaksamaan diatas setara dengan

a b c a2 + b2 + c2
+ + ≥
2n + 1 − a2n 2n + 1 − b2n 2n + 1 − c2n 2n
atau
a2 b2 c2
     
a b c
2n
− + 2n
− + 2n
− ≥0
2n + 1 − a 2n 2n + 1 − b 2n 2n + 1 − c 2n
54 BAB 3. PENJUMLAHAN SIMETRIK DAN SIKLIS

Jadi untuk menyelesaikan soal kita harus membuktikan ketaksamaan diatas.


Untuk membuktikannya, perhatikan bahwa ketaksamaan berikut benar

a a2 2n a + a2n+2 − (2n + 1)a2


− = ≥0
2n + 1 − a2n 2n 2n(2n + 1 − a2n )

karena dengan AM-GM kita peroleh 2n a + a2n+2 ≥ (2n + 1)a2 dan sebelumnya
telah diperoleh 2n + 1 − a2n ≥ 0. Dengan cara yang serupa

b b2 2n b + b2n+2 − (2n + 1)b2


− = ≥0
2n + 1 − b2n 2n 2n(2n + 1 − b2n )

dan
c c2 2n c + c2n+2 − (2n + 1)c2
− = ≥0
2n + 1 − c2n 2n 2n(2n + 1 − c2n )
Tambahkan ketiganya diperoleh

a2 b2 c2
     
a b c
− + − + − ≥0
2n + 1 − a2n 2n 2n + 1 − b2n 2n 2n + 1 − c2n 2n

seperti yang diinginkan.

Latihan 6
1. Jika x, y, z adalah bilangan riil positif, buktikan
1 1 1 9 9 9
+ + ≥ + +
x y z 4(x + y) + z 4(x + z) + y 4(y + z) + x

2. Jika a , b dan c adalah panjang sisi - sisi segitiga buktikan

a2 − b2 − c2 + bc b2 − a2 − c2 + ac c2 − b2 − a2 + ab a+b+c
+ + ≤
b+c a+c a+b 4
3. (Moldova 1999) Untuk a, b, c bilangan riil positif, buktikan

ab bc ac a b c
+ + ≥ + +
c(a + c) a(a + b) b(b + c) a+c a+b c+b

4. Jika 0 ≤ α, β, γ < π
2 dan sin2 α + sin2 β + sin2 γ = 1 maka buktikan

tan α + tan β + tan γ ≥ cos2 α + cos2 β + cos2 γ


3.2. SCHUR & MUIRHEAD 55

3.2 Schur & Muirhead


Ketaksaamaan Schur dan Teorema Muirhead walaupun penggunaannya tidak
begitu elegan tapi sangat ampuh untuk menyelesaikan persoalan pertidaksamaan,
kedua teorema tersebut sering digunakan dengan sebuah metode natural dalam
Problem Solving yang disebut Brute Force yang berarti sesuai namanya yaitu
memecahkan masalah dengan mencoba semua kemungkinan atau dalam per-
tidaksamaan ini berarti melakukan ekspansi semua suku pada soal. Kita
P akan
melihat
P bahwa disini sangat membantu unttuk menggunakan notasi cyc dan
sym .

Teorema 3.2.1 (Ketaksamaan Schur) Misalkan x , y dan z adalah bilangan


riil positif. Untuk sebarang r > 0 berlaku

xr (x − y)(x − z) + y r (y − x)(y − z) + z r (z − y)(z − x) ≥ 0.

Dengan kesamaan terjadi jika x = y = z atau jika dua variabel sama dan yang
lainnya nol.

Bukti WLOG x ≥ y ≥ z, maka xr ≥ y r dan x − z ≥ y − z jadi xr (x − z) −


y r (y − z) ≥ 0. Diperoleh

(x − y)[xr (x − z) − y r (y − z)] + z r (z − y)(z − x) ≥ 0 Q.E.D

Ketaksamaan Schur yang sering digunakan adalah kasus r = 1 yaitu yang


berbentuk X X
x3 + xyz ≥ x2 y
cyc sym

dan kasus r = 2 yang berbentuk


X X
x4 + x2 yz ≥ x3 y
cyc sym

Contoh berikut memperlihatkan penggunaannya. Meskipun tidak elegan


ketaksamaan Schur sangat ampuh, dan merupakan langkah ”awal” jika teorema
yang lain gagal.

Contoh 3.2.1 (Iran 1996) Untuk x, y dan z bilangan riil positif berlaku
 
1 1 1 9
(xy + yz + xz) + + ≥
(x + y)2 (x + z)2 (y + z)2 4

Solusi Semua materi dari subbab sebelumnya gagal. Kita paksa dengan brute
force, samakan penyebut diperoleh
!
X
2 2 9
(xy + yz + xz) (x + y) (x + z) ≥ (x + y)2 (y + z)2 (x + z)2
cyc
4
56 BAB 3. PENJUMLAHAN SIMETRIK DAN SIKLIS

setara dengan
!
X
2 2 9 2
(xy+yz+xz) (x + xy + yz + xz) ≥ ((x + y + z)(xy + yz + xz) − xyz)
cyc
4

Ekspansi semua bentuk kuadrat kita peroleh bentuk


X
4x5 y − x4 y 2 − 3x3 y 3 + x4 yz − 2x3 y 2 z + x2 y 2 z 2 ≥ 0
sym

untuk membuktikan ini kita pecah menjadi


X
x4 yz − 2x3 y 2 z + x2 y 2 z 2 ≥ 0 (∗)
sym

dan X
4x5 y − x4 y 2 − 3x3 y 3 ≥ 0 (∗∗)
sym

Ketaksamaan (*) diperoleh dari ketaksamaan Schur untuk r = 1 yaitu


X1 1
x(x−y)(x−z)+y(y −x)(y −z)+z(z −y)(z −x) ≥ 0 ⇔ x3 −x2 y + xyz ≥ 0
sym
2 2

yang apabila dikalikan dengan 2xyz kita dapatkan ketaksamaan (*).Untuk men-
dapatkan ketaksaman (**), pertama tama kita lakukan AM-GM dengan cara
berikut r
16 y 8
 
5 1 5 1 5 1 5 4 x 4
x y+ y x=3 x y+ y x≥4 = x4 y 2
3 3 3 34 3
Yang mengahsilkan
X X X
3 x5 y + y5 x ≥ 4 x4 y 2
sym sym sym

y5 x = x5 y maka
P P
karena sym sym
X X
x5 y ≥ x4 y 2 (∗ ∗ ∗)
sym sym

Setelah itu lakukan AM-GM dengan cara berikut


r
12 y 12
   
5 5 1 5 1 5 4 x
x y+y x=2 x y+2 y x≥4 = 2x3 y 3
2 2 24
yang menghasilkan X X
2 x5 ≥ 2 x3 y 3
sym
atau X X
3 x5 ≥ 3 x3 y 3 (∗ ∗ ∗∗)
sym

tambahkan (***) dan (****) diperoleh (**). Q.E.D


3.2. SCHUR & MUIRHEAD 57

Pada ketaksamaan Schur bentuk xr dapat diganti dengan fungsi tak-negatif


yang menaik karena jika f adalah fungsi yang menaik dan selalu positif maka
berlaku f (x) ≥ f (y) ≥ f (z) untuk x ≥ y ≥ z dan f (x)(x − z) − f (y)(y − z) ≥ 0
jadi
(x − y)[f (x)(x − z) − f (y)(y − z)] + f (z)(z − y)(z − x) ≥ 0
dan kita telah membuktikan
Teorema 3.2.2 (Perluasan Ketaksamaan Schur) Jika f adalah fungsi yang
selalu positif dan menaik di interval I, maka untuk setiap x, y, z ∈ I berlaku

f (x)(x − z)(x − y) + f (y)(y − x)(y − z) + f (z)(z − y)(z − x) ≥ 0

Dapat dilihat bahwa semakin cekung grafik fungsi f maka teorema 19.2 akan
semakin kuat, hal ini juga mengidentifikasikan bahwa untuk nilai r yang besar
ketaksamaan Schur akan lebih tajam, dalam arti ekspresi disebelah kiri semakin
mendekat ke 0.
Setelah Ketaksamaan Schur diberikan, dapat dikatakan bahwa semua materi
pertidaksamaan yang esensial untuk tingkat IMO atau kompetisi yang serupa
telah diberikan. Bagaimanapun juga dianjurkan bagi kita untuk mengenal lebih
banyak teorema lagi. Materi selanjutnya yaitu teorema muirhead memang
masih baru untuk digunakan di kompetisi, bahkan pada IMO 2005 yang lalu
ada beberapa juri yang kurang mengenal teorema ini. Jadi dianjurkan bagi
yang akan menggunakannya pada kompetisi untuk membuktikannya terlebih
dahulu dilembar jawaban. Sebelum menyatakan teorema Muirhead kita mulai
dengan sebuah definisi yang juga akan berguna pada subbab berikutnya.

Definisi Misalkan A = (a1 , a2 , · · · , an ) dan B = (b1 , b2 , · · · , bn ) adalah pasan-


gan terurut bilangan riil sehingga a1 ≥ a2 ≥ · · · ≥ an dan b1 ≥ b2 ≥ · · · ≥ bn .
Kita katakan A majorize B ditulis A  B jika dan hanya jika berlaku syarat
berikut

a1 ≥ b1
a1 + a2 ≥ b1 + b2
a1 + a2 + a3 ≥ b1 + b2 + b3
··· ···
a1 + a2 + · · · + an = b1 + b2 + · · · + bn

Kita katakan A minorize B ditulis A ≺ B jika dan hanya jika berlaku syarat
berikut

a1 ≤ b1
a1 + a2 ≤ b1 + b2
a1 + a2 + a3 ≤ b1 + b2 + b3
··· ···
a1 + a2 + · · · + an = b1 + b2 + · · · + bn .
58 BAB 3. PENJUMLAHAN SIMETRIK DAN SIKLIS

Perlu ditekankan sekali lagi bahwa tanda samadengan pada definsi sangat pent-
ing. Definsi diatas merupakan definsi majorize dan minorize untuk pasangan
terurut dengan syarat a1 ≥ a2 ≥ · · · ≥ an dan b1 ≥ b2 ≥ · · · ≥ bn , bagaimana-
pun juga kita dapat mendefinisikan majorize dan minorize untuk sebarang
pasangan terurut.

Definisi Misalkan A = (a1 , a2 , · · · , an ) dan B = (b1 , b2 , · · · , bn ) dan kemudian


misalkan X = (x1 , x2 , · · · , xn ) adalah permutasi dari A sehingga x1 ≥ x2 ≥
· · · ≥ xn dan Y = (y1 , y2 , · · · , yn ) adalah permutasi dari B sehingga y1 ≥ y2 ≥
· · · ≥ yn . Kita katakan A  B jika dan hanya jika X  Y , secara serupa A ≺ B
jika dan hanya jika X ≺ Y .

Teorema 3.2.3 Misalkan vektor atau pasangan terurut u = (a1 , a2 , · · · , an )


dan v = (b1 , b2 , · · · , bn ) jika u  v maka terdapat matriks M
 
λ1 λn ··· λ2
 λ2
 λ1 ··· λ3  
 ··· ··· ··· · · ·
 
λn−1 λn−2 ··· · · ·
λn λn−1 ··· λ1
Pn
dengan i=1 λi = 1 dan λi ≥ 0, sehingga

MuT = v

Bukti dari teorema ini membutuhkan aljabar linear universitas tingkat lanjut1 ,
dan tidak akan diberikan disini. Tapi untuk kasus n = 3 kita dapat mengunakan
aljabar biasa.

Teorema 3.2.4 (Muirhead) Misalkan pasangan terurut (a1 , a2 , a3 )  (b1 , b2 , b3 )


maka untuk setiap bilangan riil x, y dan z berlaku
X X
xa1 y a2 z a3 ≥ xb1 y b2 z b3
sym sym

Bukti Perhatikan bahwa dengan Teorema 1.9.2 kita dapatkan terdapat λ1 +


λ2 + λ3 = 1, λi ≥ 0 dengan

a1 λ1 + a3 λ2 + λ3 a2 = b1
a2 λ1 + a1 λ2 + λ3 a3 = b2
a3 λ1 + a2 λ2 + λ3 a1 = b3

1 Lihat Inequalities Hardy-Littewood Pölya


3.2. SCHUR & MUIRHEAD 59

Perhatikan bahwa dengan AM-GM terboboti kita peroleh


X X
xa1 y a2 z a3 = (λ1 + λ2 + λ3 ) xa1 y a2 z a3
sym sym
X X X
a1 a2 a3 a1 a2 a3
= λ1 x y z + λ2 x y z + λ3 xa1 y a2 z a3
sym sym sym
X X X
a1 a2 a3 a3 a1 a1
= λ1 x y z + λ2 x y z + λ3 xa2 y a3 z a1
sym sym sym
X
a1 a2 a3 a3 a1 a2 a2 a3 a1
= λ1 x y z + λ2 x y z + λ3 x y z
sym
X
≥ x(a1 λ1 +a3 λ2 +λ3 a2 ) y (a2 λ1 +a1 λ2 +λ3 a3 ) z (a3 λ1 +a2 λ2 +λ3 a1 )
sym
X
= xb1 y b2 z b3
sym

Teorema Muirhead juga berlaku untuk n variabel dengan bukti yang serupa.

Contoh 3.2.2 (Ketaksamaan Carlson) Jika a, b, c adalah bilangan riil posi-


tif buktikan r r
3 (x + y)(x + z)(y + z) xy + xz + yz

8 3
Solusi Misalkan p = xy + xz + yz, q = x + y + z dan r = xyz maka

(a + b)(a + c)(b + c) = pq − r

dan ketaksamaan menjadi r r


3 pq − r p

8 3
Pangkatkan 6 kita peroleh

27(p2 q 2 − 2pqr + r2 ) ≥ 64p3

Kemudian ekspansi semua kita peroleh


X
27x4 yz + 27x4 y 2 − 30x3 y 2 z − 5x3 y 3 − 19x2 y 2 z 2 ≥ 0
sym

Dengan Muirhead (4, 1, 1)  (2, 2, 2) maka


X X
19x4 yz ≥ 19x2 y 2 z 2
sym sym

(4, 2, 0)  (3, 3, 0) maka X X


5x4 y 2 ≥ 5x3 y 3
sym sym
60 BAB 3. PENJUMLAHAN SIMETRIK DAN SIKLIS

(4, 1, 1)  (3, 2, 1) maka


X X
8x4 yz ≥ 8x3 y 2 z
sym sym

dan (4, 2, 0)  (3, 2, 1) maka


X X
22x4 yz ≥ 22x3 y 2 z
sym sym

tambahkan semua untuk menyelesaikan soal.

Muirhead hanyalah aplikasi dari AM-GM, kita dapat menghindari pengu-


naan Muirhead dengan mengaplikasikan AM-GM beberapa kali, seperti pada
bukti ketaksamaan (**) di solusi Iran 1996.
Penggunaan Muirhead dan Schur biasanya dibarengi dengan homogenisasi.
Yang dimaksud homogenisasi adalah kebalikan dari normalisasi. Jika sebelum-
nya kita mengubah ketaksamaan yang homogen menjadi non-homogen dengan
menambah syarat, beikutnya akan diperlihatkan bahwa sebuah ketaksamaan
yang tak homogen dengan syarat dapat diubah menjadi yang homogen. Kesim-
pulannya membuat yang homogen menjadi tak homogen tidak selalu membuat
masalah lebih mudah, dalam pengunaan ketaksamaan Schur dan Muirhead kita
sering melakukan sebaliknya.
Jika x, y, z adalah bilangan riil positif dengan xyz = 1 maka buktikan

x3 + y 3 + z 3 ≥ x2 + y 2 + z 2

Dapat dilihat sisi sebelah kanan berderajat 3 sedangkan yang sebelah kiri berder-
ajat 2 , kita akan menyamakan derjatnya dengan menggunakan syarat xyz = 1,

yaitu dengan mengkalikan sisi kanan dengan 3 xyz jadi diperoleh

x3 + y 3 + z 3 ≥ x7/3 y 1/3 z 1/3 + x1/3 y 7/3 z 1/3 + x1/3 y 1/3 z 7/3

akhirnya
P dengan mengkalikan kedua ruas dengan 2 dan menggunakan notasi
sym kita peroleh
X X
x3 ≥ x7/3 y 1/3 z 1/3
sym sym

yang benar karena dari teorema Muirhead (3, 0, 0)  37 , 13 , 13 .




Berikut adalah sebuah contoh yang menggunakan homogenisasi

Contoh 3.2.3 (China TST 2006) Untuk x, y, z adalah bilangan riil positif
dengan x + y + z = 1 buktikan

xy yz xz 2
√ +√ +√ ≤
xy + yz yz + xz xz + xy 2
3.2. SCHUR & MUIRHEAD 61

Solusi Dengan melakukan homogenisasi ketaksamaan yang akan dibuktikan


dapat ditulis sebagai berikut
s
X xy 2(xy + yz + xz)2
≤1
cyc
xy + yz + xz (xy + yz)(x + y + z)2

Dengan menggunakan ketaksamaan Jensen pada fungsi konkav f (x) = x kita
peroleh
s v
xy 2(xy + yz + xz)2 2x(xy + yz + xz)
X uX
u
2
≤t
cyc
xy + yz + xz (xy + yz)(x + y + z) cyc
(x + z)(x + y + z)2

Jadi tinggal dibuktikan


X 2x(xy + yz + xz)
≤1
cyc
(x + z)(x + y + z)2

yang setara dengan


x y z (x + y + z)2
+ + ≤
x+z x+y y+z 2(xy + yz + xz)
perhatikan bahwa
x y z (x + y + z)2
+ + ≤
x+z x+y y+z 2(xy + yz + xz)
2x(xy + yz + xz) 2y(xy + yz + xz) 2z(xy + yz + xz)
⇔ + + ≤ (x + y + z)2
x+z x+y y+z
x2 z y2 x z2y x2 + y 2 + z 2
⇔ + + ≤
x+z x+y y+z 2
2 2 2
 2
y2 x z2y

x +y +z x z
⇔ ≤ x2 + y 2 + z 2 − + +
2 x+z x+y y+z
2 2 2 2
y2 x z2y
     
x +y +z x z
⇔ ≤ x2 − + y2 − + z2 −
2 x+z x+y y+z
2 2 2 4 4 4
     
x +y +z x y z
⇔ ≤ + +
2 x2 + xz xy + y 2 yz + z 2
Sedangkan dengan CS Engle dan ketaksamaan xy + yz + xz ≤ x2 + y 2 + z 2 kita
dapatkan
x4 y4 z4 (x2 + y 2 + z 2 )2 x2 + y 2 + z 2
+ + ≥ ≥ .
x2 + xz xy + y 2 yz + z 2 (x2 + y 2 + z 2 ) + xy + yz + xz 2
Contoh 3.2.4 (International Mathematical Olympiad 2000) Misalkan a, b, c
adalah bilangan riil positif dengan abc = 1 buktikan
   
1 1 1
a−1+ b−1+ c−1+ ≤1
b c a
62 BAB 3. PENJUMLAHAN SIMETRIK DAN SIKLIS

Solusi Kita akan membuat ketaksamaan diatas menjadi homogen, dengan cara
melakukan subtitusi a = xy , b = yz dan c = xz akan diperoleh

(x + y − z)(y + z − x)(x + z − y) ≤ xyz

yang setara dengan X X


x3 + xyz ≥ x2 y
cyc sym

ini merupakan ketaksamaan Schur untuk r = 1.

Latihan 7
1. (Japan 1997) Jika a, b, c adalah bilangan riil positif. Buktikan

(b + c − a)2 (c + a − b)2 (a + b − c)2 3


2 2
+ 2 2
+ 2 2

(b + c) + a (c + a) + b (a + b) + c 5

2. (Bulgaria 1997) Jika a, b, c adalah bilangan riil positif dengan abc = 1.


Buktikan
1 1 1 1 1 1
+ + ≤ + +
a+b+1 a+c+1 b+c+1 2+a 2+b 2+c
3. (Vasile Cartoaje) Jika a, b, c adalah bilangan riil dengan a2 + b2 + c2 = 1.
Buktikan
1 1 1 9
+ + ≤
1 − ab 1 − bc 1 − ac 2
4. (LOGIKA 2006) Jika x, y, z adalah bilangan riil positif buktikan
1 1 1 27 36 36 36
+ + + ≥ + +
x y z x+y+z 4(x + y) + z 4(x + z) + y 4(y + z) + x

5. (Hojoo Lee) Jika a, b, c adalah bilangan riil positif dengan ab + ac + bc = 1


buktikan
1 1 1 5
+ + ≥
a+b a+c b+c 2
Bab 4

Mixing Variabel dan


Metode Kalkulus

Sebuah metode yang akan kita pelajari berikutnya adalah sebuah metode yang
biasanya dipakai apabila Scur atau Muirhead gagal. Metode ini simple tapi am-
puh, yang dibutuhkan adalah kemampuan memanipulasi. Kita akan langsung
mulai dengan sebuah contoh.

Contoh 4.0.5 Jika a, b, c adalah bilangan riil positif dengan abc = 1 maka
buktikan
a3 + b3 + c3 + 9 ≥ 4(ab + ac + bc)

Solusi Kita akan menggunakan metode Mixing variabel, idenya adalah dengan
pertama - tama membuktikan bahwa ketaksamaan berlaku jika dua variabel
kita samakan, dalam kasus ini karena terdapat syarat abc = 1, kita ganti dua
buah variabel dengan variabel lain yang mempunyai hasil kali yang sama tapi
hasil penjumlahannya lebih kecil.
Misalkan f (a, b, c) = a3 +b3 +c3 +9−4(ab+ac+bc).
√ Dan WLOG a = min(a, b, c),
kemudian ganti variabel b dan c dengan t = bc, perhatikan bahwa dengan
penggantian ini kita tetap memiliki at2 = abc = 1, keuntungannya adalah
kita memperoleh b + c ≥ 2t yang dapat menaikkan atau menurunkan fungsi
f . Langkah selanjutnya adalah membuktikan

f (a, t, t) ≥ 0

Dimana
1 2
f (a, t, t) = f 1/t2 , t, t = 6 2 t7 − 2 t5 + 5 t4 + 4 t3 + 3 t2 + 2 t + 1 (t − 1)
 
t
yang jelas tidak negatif. Langkah selanjutnya adalah membuktikan f (a, b, c) ≥
f (a, t, t), yaitu membuktikan
√ √
f (a, b, c) − f (a, t, t) = b3 + c3 + 8a bc − ( bc)3 − 4a(b + c) ≥ 0

63
64 BAB 4. MIXING VARIABEL DAN METODE KALKULUS

Yang setara dengan


√ √
b3 + c3 + 8a bc ≥ ( bc)3 + 4a(b + c)
√ √
⇔ (b + c)3 − 3bc(b + c) − 4a(b + c) ≥ 2( bc)3 − 8a bc

⇔ (b + c)[(b + c)2 − 3bc − 4a] ≥ 2 bc(bc − 4a)

⇔ (b + c)[b2 + c2 − bc − 4a] ≥ 2 bc(bc − 4a)

Ketaksamaan yang terakhir benar karena dengan AM-GM b+c ≥ 2 bc dan juga
b2 + c2 − bc − 4a ≥ bc − 4a juga dengan AM-GM. Jadi kita telah membuktikan

f (a, b, c) ≥ f (a, t, t) ≥ 0 Q.E.D

Secara umum metode mixing variabel berkerja sebagai berikut:

• Pertama - tama bentuk fungsi f (x1 , x2 , · · · , xn ).

• Jika ketaksamaan simetrik maka untuk mempermudah asumsikan x1 ≥


x2 ≥ · · · ≥ xn , atau jika siklik asumsikan a = min(a, b, c).

• Lalu buktikan f (x1 , t, · · · , t) ≥ 0 atau f (x1 , t, · · · , t) ≤ 0 dengan t adalah


fungsi dalam variabel lain, jika soal mempunyai terdapat syarat, maka t
harus dibuat sedemikan rupa sehingga √ syarat tetap terpenuhi.√ Contoh
jika syaratnya abc = 1 kita coba t = bc, jika abcd = 1 coba 3 bcd, jika
a + b + c = 1 coba b+c2 . Tentu saja ini tergantung masalahnya.

• Kemudian buktikan f (x1 , x2 , · · · , xn ) ≥ f (x1 , t, · · · , t) atau ≤.

Cara diatas adalah mixing variabel yang biasa digunakan, kasus samadengan
pada mixing variabel seperti diatas tergantung pada teorema yang digunakan
pada waktu membuktikan f (a, b, c) ≥ f (a, t, t) dan f (a, t, t) ≥ 0 kemudian kasus
tersebut ditambah lagi dengan pada saat nilai dua variabel sama.
Kadangkala dalam penggunaannya Mixing Variabel dibantu oleh aplikasi
kalkulus seperti pada dua contoh berikut. Teorema - teroema berikut meru-
pakan teorema kalkulus yang sudah familiar di Matematika SMA.

Teorema 4.0.5 (Teorema nilai Ekstrim) Misalkan f adalah sebuah fungsi


yang kontinu di interval tertutup dan terbatas [a, b] maka f mencapai nilai min-
imum atau maximum di interval tersebut.

Teorema 4.0.6 (Teorema titik Ekstrim) Misalkan f adalah sebuah fungsi


yang kontinu di interval tertutup dan terbatas [a, b] maka nilai minimum atau
maksimum tercapai pada tepat satu dari ketiga calon berikut

• Titik ujung yaitu a atau b

• Titik stationer yaitu titik-titik x0 dimana f 0 (x0 ) = 0

• Titik singular yaitu titik-titik dimana f 0 (x) tidak terdefinisi.


65

Contoh 4.0.6 Buktikan untuk setiap bilangan riil a, b, c berlaku


1 1 1 4
√ +√ +√ ≥
4a2 + bc 4b2 + ac 4c2 + ab a+b+c
Solusi Misalkan
1 1 1 4
f (a, b, c) = √ +√ +√ −
4a2 + bc 4b2+ ac 2
4c + ab a + b+c

WLOG a + b + c = 1 dan a = min(a, b, c) kita akan menunjukkan


 
1−a 1−a 2 4
f a, , = ϕ(a) = √ + √ −4
2 2 17 a2 + 1 − 2 a 4 − 6 a + 2 a2
Dapat dicek bahwa lima→0 ϕ(a) = 0 dan perhatikan bahwa
4
√ −4
4 − 6 a + 2 a2

tidak mempunyai titik ekstrim di [0, 1], faktanya titik ekstrimnya adalah a = 23
yang terletak di luar [0, 1]. Jadi titik minimumnya terdapat dititik ujung yaitu
a = 0 atau a = 1. Namun
2

2
17 a + 1 − 2 a
1
hanya mempunyai satu titik ekstrim a = 17 dan titik ekstrim ini adalah titik
maksimum, sehingga titik minimumnya juga tercapai di titik ujung yaitu a =
0 atau a = 1. Karena ϕ(a) adalah penjumlahan dari √4−6 4a+2 a2 − 4 dan
2

17 a2 +1−2 a
maka ϕ(a) juga mencapai minimum dititik ujung, dapat dicek
bahwa ϕ(0) = 0 adalah nilai minimum ϕ(a) sedangkan lima→1 ϕ(a) = ∞.
Selanjutnya akan ditunjukkan f (a, b, c) ≥ f (a, t, t) dengan t = b+c
2 . Yang setara
dengan menunjukkan
 
1 1 1 1 2
√ −√ +√ +√ −√ ≥0
2
4a + bc 2
4a + t 2 2
4b + ac 2
4c + ab 2
4t + at
Ekspresi yang berada didalam tanda kurung jelas tidak negatif karena t2 =
b+c 2

2 ≥ bc. Sedangkan sisanya setara dengan
p 2
4t2 + at ≥
√ 1 + √ 1
4b2 +ac 4c2 +ab

Sisi sebelah kanan berbentuk HM (Harmonic Mean), jadi cukup bagi kita untuk
menunjukkan sisi sebelah kiri lebih besar dari GM (Geometrik Mean) -nya. Ini
setara dengan menunjukkan
 2  !2
b+c b+c
4 +a ≥ (4b2 + ac)(4c2 + ab)
2 2
66 BAB 4. MIXING VARIABEL DAN METODE KALKULUS

Ekspansi persamaan diatas, kemudian pertimbangkan hasil ekspansi sebagai


polinomial berderajat 4 dalam b yang memiliki dua akar c. Jadi dengan pem-
faktoran ketaksamaan diatas setara dengan

a2
(b − c)2 (b2 + c2 + 6bc + − 3ab − 3bc) ≥ 0
4
yang benar karena dari asumsi a ≥ b ≥ c.

Contoh 4.0.7 (International Mathematics Olympiad 2006) Tentukan bi-


langan riil M terkecil sehingga ketaksamaan

|ab(a2 − b2 ) + bc(b2 − c2 ) + ca(c2 − a2 )| ≤ M (a2 + b2 + c2 )2

berlaku untuk sembarang bilangan riil a , b dan c

Solusi Dari solusi contoh 1.5.5 kita peroleh

|ab(a2 − b2 ) + bc(b2 − c2 ) + ca(c2 − a2 )| = |(a − b)(b − c)(a − c)(a + b + c)|


= |(a − b)(b − c)(c − a)(a + b + c)|

Misalkan x = a − b, y = b − c, z = c − a dan w = a + b + c maka x + y + z = 0


dan ketaksamaan pada soal dapat ditulis
M 2
|xyzw| ≤ (x + y 2 + z 2 + w2 )2
9
Kemudian definisikan fungsi

|xyzw|
f (x, y, z, w) =
(x2 + y 2 + z 2 + w 2 )2

Akan dicari nilai M sehingga


M
f (x, y, z, w) ≤
9
Kita akan mencari nilai maksimum dari f (x, y, z, w) kemudian dikalikan 9 untuk
mendapatkan nilai M tersebut. Sebelum menggunakan Mixing Variabel, per-
tama - tama perhatikan bahwa diantara x, y dan z pasti ada dua diantaranya
yang bertanda sama karena kondisi x + y + z = 0, jadi kasus dimana x = y,
y = z atau x = z dijamin dapat terjadi, sehingga metode mixing variabel dapat
senantiasa digunakan karena dalam menggunakan mixing variabel kita menya-
makan dua buah variabel.
Karena fungsi f simetrik, WLOG asumsikan yang bertanda sama tersebut
adalah x dan y. Kemudian misalkan t = x+y 2 , karena x + y + z = 0 maka
z = −2t. Jadi metode kita akan berlanjut dengan fungsi

|2t3 w|
f (t, t, −2t, w) = .
(6t2 + w2 )2
67

Dengan AM-GM kita dapatkan

(6t2 + w2 )2 = (2t2 + 2t2 + 2t2 + w2 )2


√4
≥ (4 8t6 w2 )2

= 32 2|wt3 |

Jadi √
|wt3 | 1 2
2 2 2
≤ √ =
(6t + w ) 32 2 16

Dengan tanda samadengan terjadi jika dan hanya jika


2
(a − c)2

x+y
2t2 = w2 ⇔ 2 = = (a + b + c)2
2 2

dan
a+c
x=y⇔b=
2
a+c (a−c)2
Subtitusikan b = 2 ke persamaan 2 = (a + b + c)2 diperoleh

2(a − c)2 = 9(a + c)2

Selanjutnya kita dapat meniru langkah seperti pada contoh√1.5.5 dimana√ diper-
oleh kesamaan terjadi jika dan hanya jika (a, b, c) = 1 − 23 2, 1 + 32 2, 1 dan


permutasinya. Jadi 9162 adalah suatu dugaan nilai M yang beralasan.
Sekarang tinggal dibuktikan
x+y
f (x, y, z, w) ≤ f (t, t, −2t, w) dengan t=
2
Yang setara dengan menunjukkan

x+y 2

(−z − y)w

|xy(−x − y)w| 2
≤ 2
(x2 + y 2 + z 2 + w2 )2
 2
2 x+y
2 + z 2 + w2

atau !2 !2
x+y
p 
|xy| 2

x2 + y 2 + z 2 + w2 x+y 2

2 2 + z2 + w2
Ini juga setara dengan
 2
p x+y p x + y 2
(x + y 2 ) + x + y (z 2 + w2 )
+ |xy|(z 2 + w2 ) ≤

2 |xy|
2 | {z } 2 2
| {z } S2 | {z } | {z }
S1 S3 S4
68 BAB 4. MIXING VARIABEL DAN METODE KALKULUS

Untuk membuktikan ketaksamaan ini, pertama-tama karena x dan y diasum-


sikan mempunyai tanda yang sama maka —x+y—=—x—+—y— dan —xy—=xy.
Kita mulai dengan menunjukkan S4 ≥ S2 sebagai berikut

x + y 2 2 |x| + |y| 2 p
2 (z + w ) =
(z + w2 ) ≥ |xy|(z 2 + w2 )
2
jadi S4 ≥ S2 . Kemudian untuk menunjukkan S3 ≥ S1 kita mulai dengan AM-
GM sebagai berikut
√ √ √
|x + y| − xy ≥ xy dan |x + y| ≥ 2 xy

Kalikan keduanya untuk mendapatkan


√ √
|x + y|2 − xy|x + y| ≥ 2xy ⇔ x2 + y 2 + 2xy − xy|x + y| ≥ 2xy
 2
≥ 2√xy x + y
x + y
⇔ (x2 + y 2 )
2 2
sehingga terbukti S3 ≥ S1 , jadi S1 + S2 ≤ S3 + S4 dan pekerjaan kita selesai.

Mixing variabel mempunyai beberapa variasi. Pada contoh-contoh diatas


kita melakukan mixing variabel dengan menyamakan beberapa variabel. Pada
contoh berikut diperihatkan bahwa terdapat penggunaan lain.
Contoh 4.0.8 (Hojoo Lee) Misalkan a, b, c adalah bilangan riil positif den-
gan ab + ac + bc = 1. Buktikan
1 1 1 5
+ + ≥
a+b b+c a+c 2
Solusi Dari Pelatihan Tim IMO Cina (Dimodifikasi)
Misalkan
1 1 1 5
f (a, b, c) = + + −
a+b a+c b+c 2
WLOG a ≤ b ≤ c, kita akan menggunakan mixing varibel dengan cara berikut
 
1
f (a, b, c) ≥ f 0, a + b,
a+b

perhatikan bahwa  pasangan terurut


 (a, b, c) memenuhi ab + ac + bc = 1, begitu
1
juga pemilihan 0, a + b, a+b dikarenakan dua hal, yang pertama karena jika
 
1
(a, b, c) diganti 0, a + b, a+b maka kondisi ab + ac + bc = 1 tetap terjaga yaitu
   
1 1
0(a + b) + 0 + (a + b) =1
a+b a+b
dan yang kedua karena kesamaan pada soal terjadi jika salah satu variabel
bernilai 0 dan yang lain samadengan 1.
69
 
1
Seperti biasa langkah awal dimulai dengan membuktikan f 0, a + b, a+b ≥ 52 .
Yang setara dengan membuktikan
1 1 5
+a+b+ 1 ≥
a+b a+b+ a+b
2
1 1
Misalkan x = a+b + (a + b), maka kita harapkan x + x ≥ 55 . Dengan AM-GM
kita perloleh
1
x=+ (a + b) ≥ 2
a+b
jadi sekarang tinggal membuktikan
1 5
≥x+ untuk x≥2
x 2
sedangkan dengan pemfaktoran kita dapatkan
 
1 5 1
x+ ≥ ⇔ x− (x − 2) ≥ 0.
x 2 2
Langkah selanjutnya adalah membuktikan f (a, b, c) ≥ f (0, a + b, t) dengan t =
1
a+b .

f (a, b, c) ≥ f (0, a + b, t)
1 1 1 1
⇔ + ≥ +
b+c a+c a+b+t t
a + b + 2c a + b + 2t
⇔ 2 ≥
c + ab + ac + bc (a + b)t + t2
a + b + 2c a + b + 2t
⇔ ≥
c2 + 1 1 + t2
a + b + 2c 1 + c2
⇔ ≥
a + b + 2t 1 + t2
⇔ (a + b + 2c)(1 + t2 ) ≥ (1 + c2 )(a + b + 2t)
⇔ 2c + t2 (a + b + 2c) ≥ 2t + c2 (a + b + 2t)
⇔ 2(c − t) + t2 (a + b) − c2 (a + b) + 2t2 c − 2c2 t ≥ 0
⇔ 2(c − t) + (t2 − c2 )(a + b) + 2tc(t − c) ≥ 0
⇔ (t − c)(−2 + (t + c)(a + b) + 2tc) ≥ 0
Karena ac + bc ≤ 1 maka t − c ≥ 0 sedangkan sisanya diperoleh sebagai berikut
(−2 + (t + c)(a + b) + 2tc) ≥ 0
2c
⇔ + c(a + b) − 1 ≥ 0
a+b
2c
⇔ + c(a + b) ≥ 1
a+b
⇔ 2c + c(a + b)2 ≥ (a + b)
⇔ (2c − a − b) + c(a + b)2 ≥ 0
70 BAB 4. MIXING VARIABEL DAN METODE KALKULUS

yang benar karena dari asumusi c ≥ a dan c ≥ b mengakibatkan 2c ≥ a + b.

Sebuah solusi dapat menjadi sangat kreatif, beberapa diantaranya tidak da-
pat diklasifikasikan dalam bentuk metode-metode yang telah kita pelajari se-
belumnya karena sangat bervariasi. Pada contoh-contoh diatas telah diperli-
hatkan bahwa suatu metode dapat menjadi lebih ampuh jika digunakan secara
kreatif. Kadang-kadang peggunaan kalkulus diperlukan walaupun lebih bersifat
teknis seperti mencari nilai maksimum atau minimum, atau mencari titik ek-
strim. Pada contoh berikut kalkulus digunakan dengan cara yang lebih esensial

Contoh 4.0.9 (MIT) Jika b > a > 0 buktikan

a+b b−a √
> > ab
2 ln b − ln a
√ q
Solusi Pertimbangkan fungsi f (t) = t − 1t − ln t, dimana t > 0. Dengan
menurunkan fungsi tersebut kita peroleh
 
1 1 1 1
f 0 (t) = + −
2 t1/2 t3/2 t

dengan mengaplikasikan AM-GM kita memperoleh


 
1 1 1 1
+ ≥ ⇐⇒ f 0 (t) ≥ 0
2 t1/2 t3/2 t

dengan tanda samadengan tercapai jika dan hanya jika t = 1. Artinya fungsi
f (t) adalah fungsi naik sehingga untuk

t > 1 =⇒ f (t) > f (1) = 0

misalkan t = ab , karena b > a maka t > 1 jadi


r r  
b a b
f (t) > 0 ⇔ − > ln
a b a
b−a √
⇔ > ab
ln b − ln a
2t−2
Sekarang misalkan g(t) = ln t − t+1 maka

1 4 (t − 1)2
g 0 (t) = − =
t (t + 1)2 t(t + 1)2

jadi g(t) adalah fungsi naik sehingga untuk

t > 1 =⇒ g(t) > g(1) = 0


4.1. REFERENSI 71

b
misalkan t = a maka t > 1 sehingga
2b
a −2
 
b
g(t) > 0 ⇔ ln − b
>0
a a +1
a+b b−a
⇔ > .
2 ln b − ln a

Latihan 7
1.(AMM Januari 2006, Aliyef Yakub) Misalkan a, b dan c adalah bilangan
riil positif dengan a + b + c = 1. Buktikan
1 1 1 25
+ + ≥
a b c 1 + 48abc

Hint: Misalkan a = min(a, b, c) maka 0 < a ≤ 31 . Lalu gunakan fungsi


f (a, b, c) = a1 + 1b + 1c + 48(ab + ac + bc) − 25.
2.(Fajar Yuliawan & Dimas Yusuf ) Temukan bilangan riil terbesar k se-
hingga untuk setiap bilangan riil positif a, b, c dengan abc = 1 berlaku

a2 + b2 + c2 + 3k ≥ (k + 1)(a + b + c)

3.(Romania 2006) Untuk bilangan riil positif a, b dan c dengan a + b + c = 3


buktikan
1 1 1
+ 2 + 2 ≥ a2 + b2 + c2
a2 b c

4.1 Referensi
www.mathlinks.ro
www.olimpiade.org
Hojoo, Lee. Topics In Inequality.
Mildorf. Inequalities.
Kiran, Keldaya. A > B.

You might also like